Environmental Geology Final

¡Supera tus tareas y exámenes ahora con Quizwiz!

Rock

+Geologic: an aggregate of a mineral or minerals +Engineering: any earth material that must be blasted to be removed

How might we reduce our emissions of carbon dioxide into the environment?

- Conserve energy to reduce our dependence on fossil fuels - Use more alternative energy sources - Store carbon in Earth's systems such as forests, soils, and in rocks below the surface of Earth - Improve engineering of fossil fuel-burning power plants - Use fossil fuels that release less carbon into the atmosphere, such as natural gas

What are the by-products of nuclear fission?

- Heat - Neutrons - Energy - Fission Fragments

What three factors must be considered when restoring any type of landscape?

- Hydrologic Processes - Soils and Rocks - Vegetation

What are associated problems or hazards of geothermal energy?

- It is a site-specific energy source and is rarely found near populated areas - Development produces considerable thermal pollution from hot wastewater - Development may adversely affect nearby geyser activity by reducing or changing the heat source driving the geysers

Which of the following is a tool for studying global change?

- Marine Sediments - Real-time Monitoring - Mathematical Models - Glacial Ice ******

What condition(s) is/are necessary for the formation of an oil and/or gas reserve?

- Organic-rich Sediments - A Reservoir by Cap Rocks - Heat - Pressure

Which has the highest hydraulic conductivity?

- Sand - Dense limestone - Gravel - Sandstone - Clay

What types of environmental impacts are associated with the use of coal?

- Subsidence - Acid Mine Drainage - Acid Rain - Acid Deposition

Why is our society beginning to seek a new energy path?

- Technology and economic opportunities are favorable for the development and use of more alternative - Renewable energy - The age of cheap oil is coming to an end - Using much less fossil fuels will help reduce environmental degradation by reducing emissions of carbon dioxide and air pollutants and by increasing security

Global warming over the last 100 years is believed to be due in large part to...

- The combustion of fossil fuels - The manufacture of steel - Growing of crops - Raising of cattle.

Seawalls reduce biodiversity because...

- they have less driftwood and stranded kelp to provide habitats for organisms living on the beach. - they cause the beach to become narrower. - they cause the offshore slope to steepen. - they squeeze life into a narrower zone.

What is the probability of a ten-year flood occurring in any one year?

0.1

What are two primary characteristics of desertification?

1. Degradation of Soil 2. Degradation of Vegetation

How big was the meteorite that apparently caused the Cretaceous-Tertiary mass extinction?

10 km

According to the continuity equation, a stream with a width of 10 meters, a depth of 5 meters, and a velocity of flow of 2 meters per second has a discharge of _______.

100 cubic meters per second

The Everglades are home to more than _______ species of plants.

11,000

Worldwide, a magnitude 7.0 to 7.9 earthquake happens ______ times on average per year.

18

According to the periodic table of elements, the atomic number of potassium is _______.

19

Japan has _______ active volcanoes.

19

When was the original Clean Water Act passed into law?

1972

__________ is any substance that, in excess, is known to be harmful to desirable living organisms.

A Pollutant

Shield Volcano

A broad, convex volcano built up by successive lava flows; the largest of the volcanoes.

Tsunami Ready

A community is considered this if they: -Establish an emergency operation center with 24-hour capability -Have ways to receive tsunami warnings from the National Weather Service, Canadian Meteorological Centre, Coast Guard, or other agencies - Have ways to alert the public - Develop a tsunami preparedness plan with emergency drills - Promote a community awareness program to educate people concerning a tsunami hazard

Ecosystem

A community of organisms and its nonliving environment in which chemical elements cycle and energy flows.

Mass Wasting

A comprehensive term for any type of downslope movement of earth materials.

Fault Segmentation

A concept recognizing that faults may be divided into specific segments depending upon their geometry, structure, and earthquake history.

What is uniformitarianism?

A concept that states that present processes operated in the past, at similar rates.

Volcanic Crisis

A condition in which a volcanic eruption or prospect of an eruption produces a crisis situation for society.

Fuel Cell

A device that produces electricity directly from a chemical reaction; commonly uses hydrogen as a fuel to which an oxidant is supplied.

The Mississippi River flood of 1993 had a recurrence interval of over 100 years. What does this mean?

A flood with a magnitude equal to or exceeding the 1993 flood can be expected statistically about every 100 years or more.

Fault

A fracture or fracture system that has experienced movement along opposite sides of the fracture.

Which of the following would be an example of point source pollution?

A pipe emptying into a river **********

What is a theory?

A possible explanation for a set of observations.

Hypothesis

A statement intended to be a possible answer to a scientific question. The best hypothesis may be tested. Often multiple hypotheses are developed to answer a particular question.

Prediction

A statement that an event with specified magnitude, such as a tsunami or flood, will happen during a particular time interval. Contrast with forecast, which provides a percent chance of something happening.

Breakwater

A structure (as a wall), which may be attached to a beach or located offshore, designed to protect a beach or harbor from the force of the waves.

Groin

A structure designed to protect shorelines and trap sediment in the zone of littoral drift, generally constructed perpendicular to the shoreline.

Salt Dome

A structure produced by upward movement of a mass of salt; frequently associated with oil and gas deposits on the flanks of a dome.

All of the following are abrupt climate changes that may occur as a result of global warming, except...

A sudden disappearance of hurricane activity ******

Soil Survey

A survey consisting of a detailed soil map and descriptions of soils and land-use limitations; usually prepared in cooperation with local government.

Mid-oceanic Ridge

A topographic high commonly found in the central part of oceans characterized by seafloor spreading. An example is the Mid-Atlantic Ridge.

Typhoon

A tropical cyclone that occurs over the Indian Ocean or western Pacific; the counterpart of a hurricane that occurs in the Atlantic.

Exponential Growth

A type of compound growth in which a total amount or number increases at a certain percentage each year, and each year's rate of growth is added to the total from the previous year; characteristically stated in terms of a particular doubling time, that is, the time in years it will take the original number to double; commonly used in reference to population growth.

Consumptive Use

A type of offstream use in which the water does not return to the stream or groundwater resource after use; the water evaporates, is incorporated into crops or products, or is consumed by animals or humans.

Karst Topography

A type of topography characterized by the presence of sinkholes, caverns, and diversion of surface water to subterranean routes.

Flash Flood

A type of upstream flood in which the floodwaters rise quickly.

Cinder Cone

A volcanic, conical hill formed by the accumulation of volcanic ash and other pyroclastic deposits.

Why is topographic monitoring sometimes useful in forecasting a volcanic eruption?

A volcano sometimes swells as lava moves into the edifice.

An El Nino even is characterized by...

A warm water current in the western equatorial Pacific Ocean that flows eastward.

Pollutant

Any substance in the environment that in excess is known to be harmful to people or other desirable living organisms.

Pollution

Any substance, biological or chemical, of which an identified excess is known to be detrimental to desirable living organisms.

Ecological Restoration

Application of ecology to restore ecosystems such as rivers, wetlands, beaches, or sand dunes that have been degraded. Also the restoration of land following activities such as mining and timber harvesting.

What percentage of New Orleans flooded after the landfall of Hurricane Katrina and the failure of levees?

Approximately 80 percent

Drainage Basin

Area that contributes surface water to a particular steam network.

Drainage Basin (Watershed)

Area that contributes surface water to a particular steam network.

Triple Junction

Areas where three tectonic plates and their boundaries join.

Beach Nourishment

Artificial process of adding sediment (sand) to a beach for recreational and aesthetic purposes as well as to provide a buffer to coastal erosion.

Why are impact craters much more common on the Moon than on Earth?

As smaller objects enter Earth's atmosphere, they tend to burn up and disintegrate before actually striking the surface; the Moon has no atmosphere, and therefore no protection.

_____________ refers to the characteristic atmospheric conditions at a particular place or region over time.

Climate

Which of the following can be considered pollutants?

Bacteria and nutrients, sediment, heat, petroleum products and heavy metals *********

Mid-ocean ridge volcanism produces what type of volcanic rock?

Basalt

Why are nearby earthquakes often described as "jolting" while distant earthquakes are described as "rolling"?

Because high-frequency waves are attenuated (weakened in force) as they move away from the epicenter.

Why are the impacts of coastal hazards so great?

Because many populated areas are located near the coasts.

You are walking around a volcanic area and you come across a rock that is black, heavy, and has large, coarse mineral grains. You conclude that it is an intrusive rock. How did you know?

Because of the large, coarse mineral grains.

Why are silicate minerals the most common minerals in the crust?

Because oxygen and silicon are the most common elements in the crust.

The amount of oxygen used for bacterial decomposition is referred to as _____________.

Biochemical Oxygen Demand (BOD)

_______ refers to the number or abundance of species in an ecosystem or ecological community.

Biodiversity

As a pollutant such as methylmercury works its way through the food chain, ___________ occurs, resulting in top level food chain members ingesting even higher concentrations of the pollutant.

Biomagnification

What type of asbestos is known to cause lung disease?

Blue Asbestos

How does a catastrophe differ from a disaster?

Damages from a catastrophe are of a magnitude that requires a long recovery period.

Proxy Data

Data that are not strictly climatic but that can be correlated with climate, such as temperature of the land or sea.

An area where there is very little concentrations of oxygen at the bottom of an area or body of water is called a _______________.

Dead Zone

Water Pollution

Degradation of water quality as measured by biological, chemical, or physical criteria.

The EPA believes which of the following contaminant(s) poses an immediate health threat if water quality standards are exceeded?

Both coliform bacteria and nitrate ********

Divergent Boundary

Boundary between lithospheric plates characterized by production of new lithosphere; found along oceanic ridges.

Covergent Boundary

Boundary between two lithospheric plates in which one plate descends below the other (subduction).

What type of river channel is characterized by multiple channels and gravel bars?

Braided

What type of channel is characterized by numerous gravel bars and islands that divide and reunite the channel?

Braided channel

Ecology

Branch of biology that treats relationships between organisms and their environments.

About 94% (by weight) of all minerals mined in the US are used for ______________.

Building Materials

What are things you should do to prepare for a flood?

Buy sandbags or flood board to block doors, check with your local flood control agency to see if your property is at risk from flooding, and find out where to turn off your gas and electricity.

Soil Fertility

Capacity of a soil to supply nutrients (such as nitrogen, phosphorus, and potassium) needed for plant growth when other factors are favorable.

All of the following gases (called greenhouse gases) trap heat in the atmosphere by absorbing some of the energy radiating from Earth's surface, except...

Carbon monoxide

The maximum number of people Earth can hold without causing prohibitive environmental degradation is called ________.

Carrying Capacity

The presence of wolves in Yellowstone National Park...

Causes elk to avoid stream-side environments.

Sandstone, like most detrital (clastic) sedimentary rocks, is made out of what?

Cemented fragments of previously existing rocks

What is a possible cause of glaciation?

Changes in Earth's orbit (Milankovitch cycles) & the position of the continents at high altitudes.

Weathering

Changes that take place in rocks and minerals at or near the surface of Earth in response to physical, chemical, and biological changes; the physical, chemical, and biological breakdown of rocks and minerals.

What would be the principal option for preventing a catastrophic meteorite impact, once a threatening object was identified?

Changing the orbit of the object with small nuclear explosions.

Volatiles

Chemical compounds, such as water (H20) and carbon dioxide (CO2), that evaporate easily and exist in a gaseous state at Earth's surface.

What is a problem with multiple instream uses of rivers and streams?

Each use requires different conditions.

What scientific discipline seeks to learn how the atmosphere, oceans, land and biosphere are linked on a global scale, and how these interact to affect life on Earth?

Earth System Science

Aquifer

Earth material containing sufficient groundwater that the water can be pumped out; highly fractured rocks and unconsolidated sands and gravels make good aquifers.

Ore

Earth material from which a useful commodity can be extracted profitably.

Land Ethic

Ethic that affirms the right of all resources, including plants, animals, and earth materials, to continued existence and, at least in some locations, continued existence in a natural state.

Why are native trees more diverse in North America than in Europe?

European trees were trapped between the ice sheet and east-west trending mountain ranges.

A rapid increase in the abundance of plant life, particularly algae, is called _____________.

Eutrophication

Which type of mineral would most likely be found in a dry lakebed?

Evaporate Deposit *******

True/False: All earthquakes occur at plate boundaries.

False

True/False: Nonpoint source pollution occurs at an identifiable location whereas point source pollution is diffuse in origin.

False

True/False: Renewable energy sources have few and generally insignificant environmental impacts.

False

True/False: Usually, there is only one wave from a tsunami, and once it recedes people may return to their homes.

False

What is a possible early warning indicator of a tsunami?

Feeling tremors of an earthquake and seeing the ocean quickly recede from the beach.

Which type of flood is produced by intense rainfall of short duration over a relatively small area?

Flash flood

Floodplain

Flat topography adjacent to a stream in a river valley, produced by the combination of overbank flow and lateral migration of meander bends.

Downstream Floods

Floods produced by storms of long duration that saturate the soil and produce increased runoff over a relatively wide area. Often are of regional extent.

Factor of Safety/Safety Factor (FS)

For slope stability defined as the ratio of resisting forces to driving forces. Greater than 1, resisting forces exceed the driving forces. Less than 1, the driving forces exceed the resisting forces, and the slope can be expected to fail.

Fluid Pressure

Force per unit area exerted by a fluid in earthquakes.

What are the characteristics of a nonconformity?

Forms between older igneous or metamorphic rocks and younger sedimentary rocks

______________ account(s) for the production of nearly 90% of energy consumed in the U.S.

Fossil Fuels

What type of geologic material would make a good aquifer?

Fractured sandstone, sand, fractured granite, and gravel.

All of the following are geologic factors necessary for the safe disposal of radioactive waste, except...

Frequent earthquakes ******

Flooding

From an environmental perspective, refers to overbank flow of rivers causing potential damage to human facilities; as a natural process, refers to overbank flows that may lead to the construction of floodplains adjacent to the river channel.

Risk

From an environmental viewpoint, risk may be considered as the product of the probability of an event times the consequences.

Fossil Fuel

Fuels such as coal, oil, and gas formed by the alteration and decomposition of plants and animals from a previous geologic time.

Runup

Furthest horizontal and vertical distance that the largest wave of a tsunami moves inland.

Nuclear Energy

Generation of electricity using a nuclear reactor.

Tectonic Framework

Geometry and spatial pattern of faults or seismic sources.

Itai-Itai is a serious chronic disease that is extremely painful and attacks bones, causing them to become so thin and brittle that they break easily. Scientists are fairly certain that Itai-Itai disease is caused by ________________.

Heavy metals, especially cadmium, concentrated in soil and rice.

Which mineral has a chemical formula of Fe2O3?

Hermatite

Rockfall

Hazardous rock falls due to weathering or erosion along fractures in steep slopes made of resistant rock.

What type of metamorphism is associated with subduction zones?

High-pressure, low-temperature metamorphism

What geological processes (relating to the rock cycle) concentrate diamonds, potassium, and bauxite respectively?

Igneous, Sedimentary, and Weathering

Atoms of the same element with a different number of neutrons are known as ____.

Isotopes

Why does groundwater move so slowly through clay?

It has low conductivity.

If water is so abundant on earth, why are water resources such a big deal?

It is not always available and abundant where people are geographically.

What happens to wastewater that has undergone secondary treatment?

It is usually discharged to surface waters.

Near the ocean, which of the following is true about the groundwater table?

It is usually inclined toward the ocean. **************

Why is drainage control sometimes an effective method of landslide control?

It reduces the amount of water in the hill slope materials.

In 1994, the comet Shoemaker-Levy 9 collided with what?

Jupiter

Reserves

Known and identified deposits of earth materials from which useful materials can be extracted profitably with existing technology under present economic and legal conditions.

Before the December 26, 2004 tsunami, the most devastating tsunami in the past 200 years was caused by what and in what year?

Krakatoa volcano, 1883

It was the failing of two levees against which body of water that caused much of the flooding of New Orleans during Hurricane Katrina?

Lake Pontchartrain

Wetlands

Landscape features such as swamps, marshes, bogs, or prairie potholes that are frequently or continuously inundated by water.

Submarine Landslides

Landsides underwater that create tsunamis.

What is riprap?

Large stones used to control lateral bank erosion

The shape of shield volcanoes is a function of _________.

Lava Viscosity

In general, it takes a _______ earthquake to create enough displacement on the seafloor to generate a damaging tsunami.

M 7.5 or greater

As the Earth warms, ____________ snowfall on Antarctica is predicted.

More

You would expect a river with variable depth, turbulence, velocity, and amount of large woody debris to have _______ biodiversity than a channelized stream.

More

Why is land-use planning typically more effective than artificial control of natural hazards?

Most hazardous natural processes are not amenable to artificial control.

All of the following statements about oil and gas distribution are true, except...

Most oil and gas fields are located away from tectonic boundaries. *******

Water Cycle (Hydrological Cycle)

Movement or transfer of water from one of Earth's storage compartments, such as the ocean, lakes, and the atmosphere, to another.

What aspect of the November 13, 1985, volcanic eruption of Nevado del Ruiz was mostly responsible for the large death toll?

Mudflow

Biofuel

Oldest fuel used by humans. Organic matter burned for fuel.

Meteor

Particle from dust to centimeters in size that is destroyed in the Earth's atmosphere (shooting star).

Comet

Particle in space composed of a rocky core surrounded by ice from a few m to a few hundred km in diameter.

Meteoroid

Particle in space from less than 10 m to larger than dust size—may form from breakup of asteroids.

Your custom home designed and situated to maximize sunlight during the cold winter months and minimize sunlight during the hot summer months. This is an example of a(n) ____________ solar energy system.

Passive

___________ is the term for the time when one-half of Earth's oil has been extracted and used.

Peak Oil

Septic Tank

Tank that receives and temporarily holds solid and liquid waste. Anaerobic bacterial activity breaks down the waste, solid wastes are separated out, and liquid waste from the tank overflows into a drainage system.

Wind Power

Technology (mostly windmills) used to extract electrical energy from the wind.

Earthflows/Mudflows

The downslope flow of saturated Earth materials.

The water management philosophy put forth by Luna Leopold focuses on what natural variability in water resources...

The natural variability in water resources.

To what does the concept of acceptable risk refer?

The risk that society or individuals are willing to endure.

Why does sea cliff erosion occur at high rates in some areas?

The sea cliff may be exposed to both wave action and terrestrial mass wasting processes.

Channel Pattern

The shape of a river channel as viewed from above ("bird's-eye view"); patterns include straight, meandering, and braided.

Silicate minerals are the most abundant of the rock-forming minerals. What is the basic building block of all silicate minerals?

The silicon-oxygen tetrahedron

Seismology

The study of earthquakes as well as the structure of Earth through the evaluation of both natural and artificially generated seismic waves.

Earth Systems Science

The study of the entire planet as a system in terms of its components.

What is Earth System Science?

The study of the entire planet as a system of interrelated components.

Why does river damming sometimes contribute to coastal erosion?

The supply of sediment to beaches is shut off.

Earth Climate System

The system consisting of the atmosphere, hydrosphere, land surface, biosphere, and cryosphere (i.e., ice, snow, and frozen ground), which are linked and often interact with each other in complex ways.

Name a geographical feature caused by glacial deposition.

The till plains of central Indiana.

Asteroid

Rock or metallic particle in space from about 10 m to 1,000 km in diameter.

Igneous Rocks

Rocks formed from solidification of magma; extrusive if they crystallize on the surface of Earth, and intrusive if they crystallize beneath the surface.

How do seismologists know Earth's outer core is liquid?

S waves are absorbed upon arrival in the outer core.

Where over pumping of groundwater occurs in coastal areas, ____________ may occur.

Saltwater Intrusion

All the following mineral resources are usually imported by the U.S., except ____________.

Sand and gravel **********

Which of the following would make a good reservoir rock for oil?

Sandstone *******

How are sea otters a keystone species in Alaska and why?

Sea otters feed on urchins which allows for the flourishing of kelp.

When sea otters are absent from a kelp forest...

Sea urchin populations increase.

All of the following statements about wastewater treatment are true except _____________.

Secondary treatment removes 99% of contaminants. *********

Mineral reserves can be created by which of the following processes?

Sedimentary, weathering, igneous, metamorphic ********

The movement of _______ reveal(s) much about the internal structure of Earth.

Seismic Waves

Tsunami

Seismic sea wave generated mostly by submarine earthquakes, but also by submarine volcanic eruptions, landslides, or impacts of an asteroid; characteristically has very long wave length and moves rapidly in the open sea; incorrectly referred to as a tidal wave.

Tropical Cyclone

Severe storm generated from a tropical disturbance; called typhoons in most of the Pacific Ocean and hurricanes in the Western Hemisphere.

In a watershed underlain by shale and sandstone, which rock type would you expect to have the higher drainage density?

Shale

What type of volcano would you expect to find at an oceanic hotspot?

Shield volcano

Tsunami Runup Map

Shows the level to which the water traveled inland.

A fault in which the sides of the fault are displaced horizontally is called what?

Strike-Slip Fault

Sea Wall

Structures constructed parallel to the coastline to help delay erosion.

How can scientists collect date about changes in atmospheric carbon dioxide concentrations 800,000 years?

Studying glacial ice.

Bt

Sub-horizon enriched in clay minerals that have been translocated downward by soil-forming processes.

Bk Horizon

Sub-horizon with a high accumulation of calcium carbonate. However, it does not dominate the horizon.

________ are caused by the collapse of caverns.

sinkholes

During the past 550 million years of Earth history there have been _______ major mass extinction events.

six

________ is the downslope movement of a coherent block of Earth material.

sliding

A(n) ______ is lethal rise in the sea surface level caused by a hurricane.

storm surge

The safety factor for landslides is...

the ratio of resisting to driving forces.

How does road construction associated with timber harvesting contribute to landslide hazards?

the roads alter the movement of surface water and groundwater

Why does sea-cliff erosion occur at high rates in some areas?

the sea-cliff may be exposed to both wave action and terrestrial mass wasting processes

The vadose zone differs from the groundwater zone in what way?

the vadose zone is unsaturated, while the groundwater zone is saturated

If the trough of a tsunami arrives at shore first, then...

there will be a strong withdrawal of water, exposing the seafloor.

Ancient impact craters on Earth are hard to identify because _______.

they are often eroded or filled with sedimentary rocks

What is the main drawback of seawalls?

they reflect wave energy back across the beach and cause erosion

What is the dominant trend in water use in the US?

water use peaked in 1980

The ______ is the time it takes for successive wave crests to pass a point.

wave period

As waves enter shallow water...

wavelength decreases and wave height increases.

Why do waves expend so much energy on a rocky headland?

waves refract toward the headland on both sides

Recurrence Interval

The time between events, such as floods or earthquakes or other natural processes. Often we are interested in the average recurrence interval, which is determined by finding the mean of a series of recurrence intervals between events.

Doubling Time

The time necessary for a quantity of whatever is being measured to double.

Peak Oil

The time when half of all oil on Earth will have been extracted.

Earthquake Segment

Those parts of a fault zone that have ruptured as a unit during historic and prehistoric earthquakes.

How long does water remain in the oceans?

Thousands of Years

What is the principal goal of beach nourishment?

To maintain a positive beach budget and alleviate the need for engineered structures.

What is the objective of the United Nations Framework Convention on Climate Change?

To produce an international agreement to reduce emissions of greenhouse gases, especially carbon dioxide.

Soil

To soil scientists: Earth material so modified by biological, chemical, and physical processes that the material will support rooted plants. To engineers: Earth material that can be removed without blasting.

What is the purpose of the Near Earth Asteroid Tracking (NEAT) system?

To study the size distribution and dynamic processes associated with near Earth objects and specifically to identify those objects with a diameter of about 1 km (3280 ft).

True/False: Humans are causing rapid extinction, leading to a reduction in biodiversity.

True

True/False: Virtually all products require the use of minerals.

True

True/False:The principle of uniformitarianism can be extended to prediction of future natural hazards.

True

Why don't sailors on boats in the deep ocean notice a tsunami passing?

Tsunamis have a small wave height in the deep ocean (less than 3 feet), plus an incredibly long wavelength (more than 60 miles), so sailors do not notice their passing relative to the regular background of surface-water waves.

Contact Metamorphism

Type of metamorphism produced when country rocks are in close contact with a cooling body of magma below the surface of Earth.

Soil Slips

Type of translational slide. Very shallow slides in soil over rock that occur parallel to the slope. The slip plane is usually above the bedrock but below the soil within a slope material known as colluvium.

Pyroclastic Activity

Type of volcanic activity characterized by eruptive or explosive activity in which all types of volcanic debris, from ash to very large particles, are physically blown from a volcanic vent.

Lateral Blast

Type of volcanic eruption characterized by explosive activity that is more or less parallel to the surface of Earth. Lateral blast may occur when catastrophic failure of the side of a volcano occurs.

Volcanic Dome

Type of volcano characterized by very viscous magma with high silica content; activity is generally explosive.

Dilate

Undergo an inelastic increase in volume.

Reclaimed Water

Water that has been treated by wastewater handling facilities and may be used for other purposes on discharge, such as irrigation of golf courses or croplands.

In-stream Use

Water that is used but not withdrawn from its source; for example, water used to generate hydroelectric power.

Mega-Tsunami

Wave that is about 100 times higher than the largest tsunami produced by an earthquake, and one that could put hundreds of millions of people at risk. (can be caused by an asteroid impact)

Why does ocean swell tend to consist of waves of similar sizes?

Waves far from their storm source tend to be sorted into groups of similar waves.

Soil Profile

Weathering of earth materials that, along with biological activity and time, produces a soil that contains several horizons distinct from the parent material from which the soil formed.

Ground rupture occurs during an earthquake as...

a near-surface fault breaks the surface.

A meteoroid is...

a small particle of an asteroid.

A tsunami warning means that...

a tsunami has been detected and is traveling across the ocean toward the area.

After an earthquake in Japan, scientists in Hawaii might have reason to forecast...

a tsunami.

A lahar is produced when...

ash and other pyroclastic ejecta become saturated with water.

A(n) _______ is composed of either rock, metal, or rocky-metal mixtures, and ranges in diameter from 10 meters to 1000 kilometers.

asteroid

Meteoroids and asteroids come from the _______, which is between the orbits of Mars and Jupiter, whereas comets come from the _______.

asteroid belt, Kuiper belt or Oort cloud

The U.S. Geological Survey reports that about half of all deaths during flash floods are related to _______.

automobiles

Which of the following is incorrect about an aquifer? a) The upper surface of an unconfined aquifer is called the water table. b) An aquifer is composed of water percolating through the vadose zone. c) An aquifer is in the zone of saturation. d) An aquifer is composed at least in part by precipitation.

b) An aquifer is composed of water percolating through the vadose zone.

Which of the following is an example of a linkage between hazardous events? a) A drought occurring in an area that is often prone to flooding b) An earthquake causing a landslide along a coastal area c) A tornado touching down in an area that has already suffered from extensive erosion d) Massive rainstorms putting out a wildfire e) All of the above

b) An earthquake causing a landslide along a coastal area

Which of the following is the largest? a) Meteor b) Asteroid c) Meteoroid d) They are all about the same size.

b) Asteroid

Which of the following is a characteristic of a normal fault? a) Hanging-wall moves up relative to foot-wall at an angle greater than 45 degrees. b) Hanging-wall moves down relative to foot-wall. c) Hanging-wall moves up relative to foot-wall at an angle less than 45 degrees. d) The sides of the fault are displaced horizontally.

b) Hanging-wall moves down relative to foot-wall.

Which of the following geological processes is the slowest? a) Coastal erosion by waves b) Incision of a river into bedrock c) Earthquake rupture d) Glacier movement

b) Incision of a river into bedrock

The use of vegetation in engineering projects to achieve specific goals such as protecting stream banks from erosion is called...

biological engineering.

An artesian well is one that...

contains water

Wave normals ______ on a rocky headland.

converge

Which of the historic events listed below was the deadliest? a) 1755 Lisbon, Portugal, earthquake, tsunami, and fire b) 1998 Papua New Guinea earthquake and tsunami c) 1960 Chilean earthquake and tsunami d) 2010 Chilean earthquake and tsunami e) 2004 Sumatran earthquake and tsunami

e) 2004 Sumatran earthquake and tsunami

The material carried in chemical solution in a stream is called the..

dissolved load.

Drainage basins are separated by the ______.

drainage divide

The boundary between drainage basins is called the...

drainage divide

The _______ is the location on the surface of Earth above the point at depth where the rocks ruptured to produce the earthquake.

epicenter

A(n) _______ is a fracture system along which rocks have been displaced.

fault

Tectonic plates move about as fast as...

fingernails grow.

What aspect of tropical cyclones tends to cause the greater death and destruction?

flooding from rain, waves, and storm surge

The _______ is the flat surface adjacent to a river channel.

floodplain

From an environmental point of view, the best approach to minimizing flood damage in urban areas is __________.

floodplain regulation

The lower portion of a slump typically evolves into a(n) _______.

flow

The potential consequences of an asteroid impact or aerial blast are devastating. However, our personal risk of being the victim of such an event is small because these events occur _______.

infrequently

______ water use does not divert or remove water from its source.

instream

Vegetation decreases runoff by...

intercepting rainfall and slowing its fall to the ground

Surface water and groundwater are...

interconnected systems

In general, the frequency of an event is _______ its magnitude.

inversely proportional to

Walter Alvarez and his team of scientists found elevated levels of _______ in a clay layer marking the K-T boundary in Italy. This provided evidence of impact by a celestial object.

iridium

______ and thermoelectric generators are the primary consumptive users of fresh water.

irrigation

An indigenous ecosystem is one that...

is not substantially impacted by humans.

______ breakers are wave types that occur on steeply sloping beach faces.

plunging

An example of sound land-use planning would be...

preserving a floodplain as a greenbelt.

Groundwater mining is accomplished by...

pumping that exceeds recharge.

The spontaneous liquefaction of clay-rich sediment is called ________.

quick clay

________ designed to provide mechanical support at the base of a slope.

retaining walls

A(n) ______ is a narrow flow of water away from the beach and into deep water.

rip current

The portion of rainfall that flows off the land and directly into a river is called _____________.

runoff

The farthest horizontal and vertical distance that the largest wave of a tsunami moves inland is called...

runup

The ratio of resisting to driving forces is called the ________.

safety factor

An outbreak of cholera after a tsunami is a _______ effect.

secondary

Why is the sediment yield an important consideration in a discussion of water supplies?

sediment is a common water pollutant

Caldera eruptions are so violent because the magma contains more _______ than other magmas.

silica

The viscosity of magma is determined by...

silica content and temperature.

Which of the following would likely serve as an aquitard?

silt

The inner core of Earth is believed to be composed of ___________.

Iron

True/False: An open system exchanges energy and materials with its surroundings.

True

In about ___________ years, the cities of San Francisco and Los Angeles will be side by side.

20 Million

According to Figure 7.3, how many people in Somalia died as a result of the 2004 Indian Ocean tsunami?

289 people

What is the approximate concentration factor that has a natural concentration (percentage) of .003 and a percentage in ore of .9?

300 (divide ore or actual amount of metal/object by the natural concentration of that metal/object)

What is the residence time (in years) for water in a small lake behind a dam if the volume of the lake is 150,000,000 cubic meters, the average input from streams is 1 cubic meter per second, and the output through the dam is 1 cubic meter per second?

4.75

What is the typical tsunami speed in the deep ocean?

450 miles per hour (720 km/hr)

A population with a 1.5 percent annual growth rate will double in about ____ years.

47

What percentage of the U.S.'s population depends on groundwater for their source of drinking water?

50%

Sedimentary rock accounts for what percentage of all rocks exposed at the Earth's surface?

75 percent

How many neutrons does carbon-14 have?

8

Law of Superposition

A fundamental law of the geological sciences that states that for any sequence of sedimentary strata that has not been overturned, the youngest rocks are at the top and the oldest at the bottom or base of the sedimentary sequence. Another way of stating it is that for any given strata or sedimentary unit, rocks above are younger and those below are older.

What is an ecological community?

A group of populations of different species living in the same area with varying degrees of interaction with each other.

Earthquake Cycle

A hypothesis to explain periodic occurrence of earthquakes based on drop in elastic strain after an earthquake and re-accumulation of strain before the next event.

Glacier

A land-bound mass of moving ice.

What caused the giant wave that occurred in Lituya Bay, Alaska, in 1958?

A landslide of rock fell from a cliff into the bay.

River

A larger stream (any body of water that flows in a channel)

Biochemical Oxygen Demand (BOD)

A measure of the amount of oxygen necessary to decompose organic materials in a unit volume of water. As the amount of organic waste in water increases, more oxygen is used, resulting in a higher BOD.

Magma

A naturally occurring molten rock material, much of which is in a liquid state.

Law of Original Horizontality

A principal law of the geological sciences that states that a sedimentary stratum, at the time it was deposited, was nearly horizontal; this does not mean that individual grains in a stratum are deposited horizontally, but that the sedimentary bed itself is essentially horizontal.

Floodplain Regulations

A process of delineating floodplains and regulating land uses on them.

Wastewater Renovation and Conservation Cycle

A process of recycling liquid waste that includes return of treated wastewater to crops or irrigation and continued renovation through recharge of groundwater; the reused part involves pumping out the groundwater for municipal, industrial, or other purposes.

Submarine Trench

A relatively narrow, long (often several thousand km), deep (often several km) depression on the ocean floor that forms as a result of convergence of two tectonic plates with subduction of one.

Bauxite

A rock composed almost entirely of hydrous aluminum oxides; a common ore of aluminum.

Metamorphic Rock

A rock formed from preexisting rock by the effects of heat, pressure, and chemically active fluids beneath Earth's surface. In foliated metamorphic rocks, the mineral grains have a preferential parallel alignment or segregation of minerals; non-foliated metamorphic rocks have neither.

Sedimentary Rock

A rock formed when sediments are transported, deposited, and then lithified by natural cement, compression, or other mechanism; detrital sedimentary rock is formed from broken parts of previously existing rock; chemical sedimentary rock is formed by chemical or biochemical processes removing material carried in chemical solution.

Modified Mercalli Scale

A scale with 12 divisions that subdivide the amount and severity of shaking and damage from an earthquake.

Rip Current

A seaward flow of water in a confined narrow zone from a beach to beyond the breaker zone.

Coal

A sedimentary rock formed from plant material that has been buried, compressed, and changed.

Soil Chronosequence

A series of soils arranged in terms of relative soil profile development from youngest to oldest.

All of the following are problems associated with the use of nuclear power, except...

A shortage of uranium resulting in high cost of energy production ******

How much more energy is released in a M 6.5 earthquake versus a M 5.5 earthquake?

About 32 times

Beach

Accumulation of whatever loose material (commonly sand, gravel, or bits of shell and so forth) that accumulate on a shoreline as the result of wave action.

Which of the following is an impact associated with mineral development?

Acid mine drainage, social impact, toxic airborne pollutants, biological degradation ******

All of the following are ways to prevent acid-rain related damage, except...

Addition of buffer material such as lime ******

Which is the most effective type of wastewater treatment?

Advanced *******

What is tephra?

All types of volcanic debris explosively ejected from a volcano.

Glaciers in mountain valleys are called what?

Alpine Glaciers

Paleomagnetism

Also known as paleomagnetism; refers to the study of magnetism of rocks and the intensity and direction of the magnetic field of Earth in the geologic past.

Law of Faunal Assemblages

Also known as the law of faunal succession, this is a general law of the geological sciences that states that the fossils or organisms succeed one another in an order that may be recognized. In other words, the fossil content of sedimentary rocks suggests the rocks' relative ages.

What is/are the most important cause(s) of river floods?

Amounts and rates of precipitation and runoff

The December 26, 2004 tsunami was triggered by what?

An Interplate Earthquake

Mineral

An element or chemical compound that is normally crystalline and is formed as the result of geologic processes.

Channelization

An engineering technique to straighten, widen, deepen, or otherwise modify a natural stream channel.

Precautionary Principle

An environmental planning tool that advocates taking cost-effective, proactive steps to eliminate or reduce the consequences of an environmental problem even if the science is not completely worked out. In simple words, better safe than sorry.

Acid Mine Drainage

An environmental problem related to the discharge of acidic waters resulting from the weathering of sulfide minerals, such as iron pyrite, associated with coal and sulfide mineralization of important metals, such as copper, silver, and zinc.

What does an orange volcano alert mean?

An eruption is likely within hours or days

What caused the tsunami of 1700?

An estimated M 9 earthquake in the Cascadia subduction zone

El Nino

An event during which trade winds weaken or even reverse and the eastern equatorial Pacific Ocean becomes anomalously warm; the westward moving equatorial current weakens or reverses.

Catastrophe

An event or situation causing sufficient damage to people, property, or society in general from which recovery and/or rehabilitation is long and involved; natural processes most likely to produce a catastrophe include floods, hurricanes, tornadoes, tsunamis, volcanoes, and large fires.

What is an aerial burst?

An explosion of a meteor entering Earth's atmosphere.

Law of Crosscutting Relationship

An igneous intrusion is always younger than the rock it cuts across and older than the ones it does not.

Climate Forcing

An imposed perturbation of Earth energy balance. Major climatic forcings associated with global warming include: greenhouse gases, such as carbon dioxide and methane; reflective aerosols in the atmosphere; and black carbon. Forcing of the climate system also includes solar activity and Milankovitch Cycles.

Mantle

An internal layer of Earth approximately 3000 km thick composed of rocks that are primarily iron and magnesium-rich silicates. The lower boundary of the mantle is with the core, and the upper boundary is with the crust. The boundary is known as the Mohorovicic discontinuity (also called the Moho).

How does an open system contrast with a closed system?

An open system exchanges energy and/or materials with its surroundings, while a closed system does not.

What is the most common rock type of composite volcanoes?

Andesite

Lava Flow

Eruption of magma at the surface of Earth that generally flows downslope from volcanic vents.

The stratospheric ozone hole occurs along the...

Antarctic Polar Vortex

Hot Spot

Assumed stationary heat source located below the lithosphere that feeds volcanic processes near Earth's surface.

Dilatancy-Diffusion Model

Assumes that the first stage in earthquake development is an increase of elastic strain in rocks that causes them to dilate after the stress on the rock reaches one-half the rock's breaking strength.

Why do some asteroids leave the asteroid belt and cross the orbits of Earth and other solar system bodies?

Asteroids collide in the asteroid belt and some asteroids are diverted into other orbits.

What are isotopes?

Atoms of the same element with different atomic masses.

All of the following are places where acid mine drainage is likely to occur, except ___________.

Calcite mines *********

What type of eruptions has only happened about 10 times in the last million years?

Caldera Eruptions

What is the principal difference between craters and calderas?

Calderas are much larger depressions created by collapse of the upper portions of the volcano.

What caused the most deaths from the 1991 Mount Pinatubo eruption on Luzon Island in the Philippines?

Collapse of buildings

Why do composite volcanoes consist of alternating lava and pyroclastic layers?

Composite cones are created by a mixture of explosive activity and lava flows.

Uniformitarianism

Concept that the present is the key to the past; that is, we can read the geologic record by studying present processes.

What were the Appalachian Mountains formed from?

Continent-continent Plate Collision

Alfred Wegener first presented evidence for his hypothesis called _______ in 1915.

Continental Drift

Most scientists believe ____________ is the force that drives plate tectonics.

Convection

Subduction Zone

Convergence of tectonic plates where one plate dives beneath another and is consumed in the mantle.

Desertification

Conversion of land from a more productive state to one more nearly resembling a desert.

You are in the field working on your first geology project for class. You come across a series of layered rocks and take note of their location on your map. You see a line of rock cutting through the layers and instantly realize that it must be younger than the layered rock. You just used the law of _______.

Crosscutting Relationships

How do point bars and cut banks differ from one another in a stream system?

Cut banks are sites of erosion, while point bars are sites of deposition.

Which of the following is a way to make nonrenewable mineral resources more "sustainable"?

Developing more efficient ways of mining mineral resources, recycling existing forms, developing more efficient ways of exploring for mineral resources, finding more sustainable substitutes for nonrenewable mineral resources. *********

Sustainable Energy Policy

Development of energy policy that finds useful sources of energy that do not have adverse _environmental effects or minimizes those effects in such a way that future generations will have access to energy resources and a quality environment.

What is sustainability?

Development that ensures that future generations will have equal access to the resources that our planet offers.

Nonpoint Source

Diffused and intermittent sources of air or water pollutants.

When the availability of a particular mineral becomes a limitation, which of the following represent(s) (a) possible solution(s)?

Do without, use less, find a substitute, recycle what has already been obtained, find more sources.

A watershed, or _________, is an area of land that contributes water to a particular stream or river.

Drainage Basin

What is the most common potential cause of a tsunami?

Earthquake

What is not one of the three methods of finding out the average recurrence interval of earthquakes?

Earthquake Segments

Slow Earthquake

Earthquake produced by fault rupture that can take days to months to complete.

Why have twentieth-century earthquakes in Turkey suggested that some earthquakes beget subsequent earthquakes?

Earthquakes have occurred in a pattern from east to west.

The society of ________ Island was destroyed by environmental degradation.

Easter

_______ is the study of controls over the distribution and abundance of living things.

Ecology

A(n) _______ is an ecological community and its nonliving environment in which energy flows and chemicals (such as nutrients and water) cycle.

Ecosystem

____________ entails designing and using equipment that yields more power from a given amount of energy, while wasting less energy.

Efficiency

____________ streams tend to flow all year.

Effluent

Rocks on either side of a fault segment may be strained due to stress. When the deformed rocks finally rupture, the stress is released, and the sides of the fault "snap" into their new position. What is this called?

Elastic rebound

Darcy's Law

Empirical relationship that states that the volumetric flow rate such as cubic meters per day is a product of hydraulic conductivity, hydraulic gradient, and cross-sectional area of flow; developed by Henry Darcy in 1856.

Solar Energy

Energy that is collected from the Sun.

Desalination

Engineering processes and technology that reduce salinity of water to such a level that it may be consumed by people or used in agriculture.

The principle of ________ states that one action leads to subsequent actions in linked systems.

Environmental Unity

What caused the 1883 tsunami in the Sundra Strait between Java and Sumatra?

Explosion of Krakatoa volcano and sudden collapse into the ocean

Which step(s) of mineral development have/has the greatest impact on the environment?

Extraction and processing ********

In Yosemite National Park, strong bedrock influences the formation of what types of hill slopes?

Glacial

How might global warming exacerbate coastal hazards?

Global warming will likely be accompanied by sea-level rise.

What aspect of hillside development has contributed to the frequency of landslides in southern California?

Grading of hill slopes to create pads for home construction.

Why does population increase affect the number of catastrophic events?

Greater numbers of people occupy marginal lands in the path of hazardous processes.

The term ___________ refers to the trapping of heat by Earth's atmosphere.

Greenhouse Effect

What is the name for any process that adds water to an aquifer?

Groundwater Recharge

What happens when water is pumped from a well?

Groundwater is discharged, a cone of depression forms, and the water table is lowered.

What is not a hazard created by ash eruptions?

Groundwater may be contaminated by sediment.

Why is it better to prevent groundwater pollution before it happens?

Groundwater pollution is difficult to detect and treat.

What is the definition of groundwater mining?

Groundwater pumping from an aquifer at a rate greater than replenishment.

Rock Cycle

Group of processes that produce igneous, metamorphic, and sedimentary rocks.

Fault Zone

Group of related faults roughly parallel to each other in map view. They often partially overlap or form braided patterns. The width varies ranging from a meter or so to several kilometers.

What is exponential growth?

Growth that occurs as a constant percentage of the existing amount.

You observe that a bird lives in and around your backyard and eats berries and worms. Your yard is its _______ and eating berries and worms is its _______.

Habitat, Niche

Previous environmental degradation led to catastrophe in the 2010 earthquake in ________.

Haiti

K Horizon

Horizon completely dominated by calcium carbonate.

O Horizon

Horizon composed mostly of organic materials including decomposed or decomposing leaves, twigs, etc. The color of the horizon is often dark brown or black.

R Horizon

Horizon composed of unweathered (unaltered) parent material.

A Soil Horizon

Horizon is composed of both mineral and organic materials. The color is often light black to brown. Leaching occurs here and moves clay ad other materials such as iron and calcium to the B horizon.

C Horizon

Horizon is composed of partially altered (weathered) parent material; rock is shown here but the material could also be alluvial in nature, such as river gravels in other environments. The horizon may be stained red with iron oxides.

B Horizon

Horizon is enriched in clay, iron, oxides, silica, carbonate, or other material leached from overlying horizons. Horizon is known as the zone of accumulation.

E Horizon

Horizon that is composed of light-colored materials resulting from leaching of clay, calcium, magnesium, and iron to lower horizons. Combined with the A Horizon constitutes the zone of leaching.

Hydrothermal

Hot Water

The Hawaiian Islands formed as a result of the Pacific plate moving over a _______.

Hotspot

The ability of a particular material to allow water to move through it is called _____.

Hydraulic Conductivity

The rate of groundwater flow is proportional to the...

Hydraulic Gradient and Hydraulic Conductivity.

Oil and gas are known as __________.

Hydrocarbons

Fault-Valve Mechanism

Hypothesizes that fluid (usually water) pressure rises until failure occurs, thus triggering an earthquake, along with upward fluid discharge.

Future use and environmental impacts of coal include all of the following, except...

Impacts will decrease as the US will run out of coal soon ******

The United States experiences eruptions two or three times a year. Where do most of them occur?

In Alaska

Biodiversity

In general, refers to the variety of life in an area, region, or Earth. Also refers to the total number of species (richness) or the main species encountered (dominance).

Of the following choices, the best place for a geothermal energy plant would be ___________.

In the region of Yellowstone National Park ******

Wadati-Benioff Zone

Inclined zone of earthquakes produced as a tectonic plate is subducted.

Resource

Includes reserves plus other deposits of useful earth materials that may eventually become available.

What effect does a land-use change from forest to agriculture near a stream have on the stream?

Increase in channel slope and velocity of the water

The Medieval Warm Period corresponds with a time of...

Increased Solar Activity

If a stream channel is of constant width and depth, how must increased discharge be accommodated?

Increased Velocity

Why might logging of a drainage basin change the slope of the river?

Increased sediment load causes the river to steepen its gradient to transport more sediment.

The average density of Earth's layers ___________ toward the center of the planet.

Increases

Which two plates were directly involved in the earthquake that triggered the December 26, 2004 tsunami?

Indian and Australian

________ water use does not divert or remove water from its source.

Instream

Beach Budget

Inventory of sources and losses of sediment to a particular stretch of coastline.

What is an atom called after it gains or loses electrons and is no longer neutral?

Ion

What element was found to be enriched in a thin clay layer marking the Cretaceous-Tertiary boundary at several sites around the world?

Iridium

All through high school, you throw your reports and term papers into a box as you get them back from your teachers. After you graduate, you discover that the older papers are on the bottom and the newer papers are on top. This is an example of what geologic principle?

Law of Superposition

Soil Horizons

Layers in soil (A, B, C, etc.) that differ from one another in chemical, physical, and biological properties.

Metallic minerals are consumed in _____________ amounts than nonmetallic minerals and materials.

Lesser

What event with a low catastrophe potential kills a large number of people each year?

Lightning

_______ is the name for the transformation of water-saturated granular material from a solid to a liquid state.

Liquefaction

Slip Rate

Long-term rate of slip (displacement) along a fault; usually measured in millimeters or centimeters per year.

Addition of Volatiles

Lowers the melting temperature of rocks by helping to break chemical bonds within silicate minerals.

The water management philosophy put forth by ______ focuses on natural variability in water resources.

Luna Leopold

______ proposed a new philosophy of water management, based on the acknowledgement that groundwater and surface water are subject to natural variations.

Luna Leopold

The ___________ is nearly 3000 kilometers thick and is mostly solid, with an average density of 4.5 grams per cubic centimeter.

Mantle

Shake Map

Map showing pattern and extent of seismic shaking from an earthquake.

A comprehensive term for any type of downslope movement of Earth materials is ____________.

Mass Wasting

A comprehensive term for any type of downslope movement of Earth materials is...

Mass wasting

Which of the following is a mineral resource that is concentrated at divergent plate boundaries?

Massive sulfide deposits *********

Evaporite

Material with a high commercial value found in evaporated lakes. Grouped into 3 types: 1. Marine (Solids)- potassium and sodium salts, calcium carbonate, gypsum, and anhydrite 2. Nonmarine (Solids)- sodium and calcium carbonate, sulfate, borate, nitrate, and limited iodine and strontium compounds 3. Brines (Liquids derived from wells, thermal springs, and inland salt lakes, and seawaters)- bromine, iodine, calcium chloride, and magnesium

Shrink-Swell Potential

Measure of a soil's tendency to increase and decrease in volume as water content changes.

In what type of bond are electrons shared between all atoms in a solid mass?

Metallic Bond

Black smokers produce what type of mineral?

Metallic Sulfides

What type of rock forms from alteration by heat, pressure, or chemically active fluids?

Metamorphic Rocks

Coal beds in the wester US are now being mined to extract ____________.

Methane

Coal-Bed Methane

Methane stored on surfaces of organic matter in coal.

Tight Gas

Methane that is distributed in small tight openings throughout black shale (commonly found 1 km beneath N Appalachia). An unconventional reservoir compared to conventional natural gas reservoirs where methane is in pockets related to geologic structure.

The process of converting an old, unsightly mining site into an environmentally safe, clean, and useful area is called _____________.

Mine Reclamation

Ash Flow

Mixture of volcanic ash, hot gases, and fragments of rock and glass that flows rapidly down the flank of a volcano; may be an extremely hazardous event.

Colluvium

Mixture of weathered rock and other material.

Lava

Molten material produced from a volcanic eruption, or rock that forms from solidification of molten material.

Levee

Natural levees result from overbanked flows of rivers; human-constructed levees are earthen embankments along a river channel to protect land adjacent to the river from flooding.

Paleo-Proxy Data

Natural records of climate variability as indicated by tree rings, ocean sediments, ice cores, fossil pollen, corals, and carbon-14.

Earthquake

Natural shaking or vibrating of Earth in response to the breaking of rocks along faults. The earthquake zones of Earth generally are correlated with lithospheric plate boundaries.

Where would you expect to find mercury deposits?

Near convergent plate boundaries.

The increase in atmospheric aerosols after a volcanic eruption leads to a ____________ climate forcing.

Negative

Manganese Oxide Nodules

Nodules of manganese, iron with secondary copper, nickel, and cobalt, which cover vast areas of the deep-ocean floor.

Which of the following is considered the necessary concentration factor for metallic minerals?

None of the above **********

The ages of the Hawaiian Islands increase to the northwest, telling us that the Pacific plate is moving to the _____________.

Northwest

Tsunami Watch

Notification that an earthquake that can cause a tsunami has occurred.

A mineral resource is a concentration of a naturally occurring material that can _____________ be extracted at a profit.

Now or Potentially

All of the following are considered renewable energy sources, except...

Nuclear ********

The positively charged, dense center of an atom is called the _______.

Nucleus

Decompression Melting

Occurs when the overlying pressure exerted on hot rock within asthenosphere is decreased

What type of plate boundary are the Andes Mountains and the Peru-Chile Trench associated with?

Ocean-continent convergent plate boundary

Jetty

Often constructed in pairs at the mouth of a river or inlet to a lagoon, estuary, or bay, designed to stabilize a channel, control deposition of sediment, and deflect large waves.

On what type of Earth materials do any kind of slumps, slips, rockfalls, etc. typically happen?

On slopes made of shale or built on weak volcanic pyroclastic materials.

In a meandering stream, where does erosion take place most rapidly?

On the outsides of bends

What does the concept of environmental unity state?

One action leads to subsequent actions in linked systems.

Land Use Planning

One of the most environmentally sound adjustments to hazards. People can avoid building on floodplains, in areas where there are active landslides, or in places where coastal erosion is likely to occur.

P Wave (Compressional Waves)

One of the seismic waves produced by an earthquake; the fastest of the seismic waves, it can move through liquid, solid, and gaseous materials. Moves much more slowly through liquids, though. When propagated into the atmosphere, some people can hear the earthquake before they can feel it.

Natural Disaster

One or more of the following criteria are met: a) 10 or more people are killed b) 100 or more people are affected c) a declaration of emergency is issued d) a request is made for international assistance

Surface Wave (R Waves)

One type of wave produced when S and P waves reach the surface; generally cause most of the damage to structures on the surface of Earth. Slowest of all the wave types. Complex horizontal and vertical ground movement or a rolling motion which can crack walls and the foundation of many things.

A(n) ________ system is one that exchanges energy and/or materials with its surroundings.

Open

The term ____________ refers to useful metallic minerals that can be mined at a profit.

Ore

Placer Deposit

Ore deposit found in material transported and deposited by such agents as running water, ice, or wind; for example, gold and diamonds found in stream deposits.

Lithosphere

Outer layer of Earth, approximately 100 km thick, that comprises the plates that contain the ocean basins and continents.

_______ can travel through solid, liquid, and gaseous materials, whereas _______ can travel only through solid materials.

P -waves, S waves

Surface waves are produced by...

P and S waves reaching the surface.

Surface waves are produced by...

P waves and S waves reaching the surface.

Damaging tsunamis in historic times have been mostly where?

Pacific Basin

What was the name of the last supercontinent to exist on Earth?

Pangaea

Meteorite

Particle from dust to asteroid size that impacts Earth's surface.

In what way is the rock cycle intertwined with plate tectonics?

Plate tectonic processes control the rock-forming processes prevalent at plate boundaries.

Conservation

Policy for resources such as water and energy that moderates or adjusts demands in order to minimize expenditure of the resource; may mean getting by with less through improved technology to provide just the amount of the resource necessary for a given task.

Which of the following statements about water pollution is correct?

Pollution is a resource out of place. ***************

Sediment Pollution

Pollution of some part of the environment either on land or in a body of water by sediment that has been transported into that environment by wind or water; an example is turbidity of a water supply (muddy water).

What is the most common cause of septic system failure?

Poor Soil Drainage

When glacial ice melts and bare ground is exposed, there is a ____________ feedback because white ice reflects sunlight and darker rock absorbs more sunlight.

Positive

Water Management

Practice of managing our water resources.

Which type of wastewater treatment uses screening to remove grit?

Primary **********

Before defining the objectives and goals of a restoration project, planners must...

Provide a clear understanding of the need for the restoration.

Some volcanoes produce fast-moving avalanches of hot ash, rock, volcanic glass fragments, and gas. What are these avalanches called?

Pyroclastic flows

On what basis are Mercalli intensity values assigned to locations?

Qualitative perceptions of and structural response to the shaking.

Acid Rain

Rain made artificially acidic by pollutants, particularly oxides of sulfur and nitrogen; natural rain water is slightly acidic owing to the effect of carbon dioxide dissolved in the water.

Debris Flow (Mudflow/Lahar)

Rapid downslope movement of Earth material often involving saturated, unconsolidated material that has become unstable because of torrential rainfall.

Snow Avalanche

Rapid downslope movement of snow, ice, and rock.

Cultural Eutrophication

Rapid increase in the abundance of plant life, particularly algae, in freshwater or marine environments resulting from input of nutrients from human sources to the water.

Water that has undergone advanced treatment is known as ______________.

Reclaimed Water

Large-scale ________ of Earth materials will be required to meet future resource demands.

Recycling

All of the following are methods to reduce the mineral development impacts from operating mines, except _________.

Recycling ******

Alternative Energy

Refers to energy sources that are alternative to the commonly used fossil fuels.

Relative Profile Development

Refers to soils that may be weakly, moderately, or strongly developed, depending on specific soil properties.

Ozone Depletion

Refers to stratospheric loss of ozone, generally at the South Pole, related to release of chlorofluorocarbons (CFCs) into the atmosphere.

Environmental Crisis

Refers to the hypothesis that environmental degradation has reached a crisis point as a result of human use of the environment.

Global Warming

Refers to the hypothesis that the mean annual temperature of the lower atmosphere is increasing as a result of burning fossil fuels and emitting greenhouse gases into the atmosphere.

Material Amplification

Refers to the phenomenon that some Earth materials will cause the amplitude of seismic shaking to increase. This is generally associated with soft sediment, such as silt and clay deposits.

What was not a factor in the flooding of New Orleans during Hurricane Katrina?

Regional subsidence was slower than was appreciated.

From the environmental standpoint, what is the best method for maximal development of floodplains with minimal damage by flooding?

Regulate development.

Caldera Eruption

Relatively infrequent large volcanic eruption that is associated with a catastrophic explosion that may produce a very large volcanic crater 30 or more kilometers in diameter.

What is he fundamental problem associated with mineral resource availability?

Reserves are difficult to maintain.

You are working in the field as a geologist. You are mapping a large iron ore deposit. After you identify the amount of ore that is currently available, you speculate that there is a vast volume in the same area that could be extracted with the use of technology not yet developed. This part of the deposit is a ______________.

Resource

In what way was the 1845 mudflow on Nevado del Ruiz responsible for the huge amount of lives lost in the same area in 1985?

Rich soils deposited by the original mudflow resulted in an agricultural center being developed, drawing a large number of people to inhabit the area.

Why are iron and steel recycled in large volumes?

Significant environmental impacts related to disposal of over 50 million tons of iron and steel would result if we did not recycle, the market for iron and steel is huge, and there is a large scrap collection and processing industry, an enormous economic burden would result if recycling was not done.

What materials would you expect to find on a floodplain?

Silt and sand

Subsidence

Sinking, settling, or other lowering of parts of the crust of Earth.

What driving force of plate movement is likely dominant?

Slab Pull

Rotational Slide (Slump)

Sliding occurs along a curved slip surface

Global Dimming

Slight cooling caused by human release of air pollution particles that reflect incoming solar radiation back to space.

What strategy has been suggested to stabilize the La Conchita, California, landslide?

Slope Grading

Why are landslides more common on steep slopes?

Slope angle determines the relative magnitude of driving forces

Tectonic Creep

Slow, more or less continuous movement along a fault.

How do slumps and translational slides differ?

Slumps are rotational, while translational slides are planar.

Natural Gas

Sometimes also referred to as natural energy gas or hydrocarbons that include ethane, propane, butane, and hydrogen.

Landslide

Specifically, rapid downslope movement of rock and/or soil; also a general term for all types of downslope movement.

New crustal material is formed at what location?

Spreading Centers

Tsunami Warning

Statement that a tsunami has been detected and is spreading across the ocean toward their area.

Sea Cliff

Steep (commonly near-vertical) bluff adjacent to and adjoining a beach or coastal environment; produced by a combination of erosional processes including wave activity and subaerial processes such as weathering, landsliding, and runoff of surface water from the land.

Composite Volcano

Steep-sided volcanic cone produced by alternating layers of pyroclastic debris and lava flows.

The law of _______ states that if a series of layered sediments have not been overturned, the oldest layers are on the bottom and the youngest are on top.

Superposition

Sinkhole

Surface depression formed by solution of limestone or collapse over a subterranean void such as a cave.

Spreading Center

Synonymous with mid-oceanic ridges where new crust is continuously added to the edges of lithospheric plates.

Transform Boundary

Synonymous with transform faults, occurring where edges of two plates slide past one another; most are boundaries within oceanic crust; an example on land is the San Andreas fault in California.

Forty-one people were killed by the collapse of a tiered freeway during the 1989 Loma Prieta earthquake near San Francisco. What caused this section of freeway to collapse when other parts of the freeway in the same area survived?

That section of freeway was constructed on bay fill and mud.

Virtual Water

The amount of water necessary to produce a product, such as rice or, in industry, an automobile.

Earth System Science

The attempt to understand how the Earth works as a system. They do this by learning how the various components of the system-the atmosphere, oceans, land, and biosphere-are linked on a global scale and interact to affect life on Earth.

You are working as a naturalist observing the interactions of animals in a mountain park. Over several years of work, you notice that when the population of a particular beetle is low, there are more individuals of a certain fish species in the rivers. What might this indicate about the beetle?

The beetle is a keystone species.

Why were most of the 2009 Italian earthquake deaths and 2010 Haitian Earthquake deaths human-caused?

The buildings were not designed to withstand shaking and they collapsed, killing people inside.

Climate

The characteristic atmospheric condition (weather) at a particular place or region over time periods of seasons, years, or decades.

Wilson Cycle

The cyclic nature of ocean basins; named after John Tuzo Wilson.

Why does withdrawal of fluids sometimes lead to subsidence?

The fluid pressure provides partial support for the overlying rock mass.

Why is the geologic history of a volcano important in hazard prediction?

The geologic history reveals the frequency and style of eruptions.

Design Basis Ground Motion

The ground motion that has a 10 percent chance of being exceeded in a 50 year period.

What is the principal difference between Earth's inner core and outer core?

The inner core is solid, while the outer core is liquid.

What can commonly happen in dead zones?

The low oxygen commonly kills shellfish and crabs.

Moment Magnitude

The magnitude of an earthquake based on its seismic moment, which is the product of the average amount of slip on the fault that produced the earthquake, the area that actually ruptured, and the shear modulus of the rocks that failed.

Carrying Capacity

The maximum number of a population of a species that may be maintained within a particular environment without degrading the ability of that environment to maintain that population in the future.

What happened to the Trans-Alaskan pipeline during the 2002 Alaskan earthquake?

The pipeline adjusted to the displacement by moving its Teflon shoes along slider beams.

Epicenter

The point on the surface of Earth directly above the hypocenter (area of first motion) of an earthquake.

Focus

The point or location in Earth where earthquake energy is first released; during an earthquake event, seismic energy radiates out from the focus.

Leaching

The process of dissolving, washing, or draining Earth materials by percolation (filtered or drained through porous surface or substance) of groundwater to other liquids.

Channel Restoration

The process of restoring stream channels and adjacent areas to a more natural state.

Discharge

The quantity of water flowing past a particular point on a stream, usually measured in cubic feet per second (cfs) or cubic meters per second (cms).

Geothermal Energy

The useful conversion of natural heat from the interior of Earth.

In what way does the vadose zone differ from the groundwater zone?

The vadose zone is unsaturated while groundwater is saturated.

If the discharge of a stream is constant, but the cross-sectional area of the stream decreases, what happens to the velocity of the water?

The velocity of the water increases.

Why might the infiltration of water deep into a slide mass increase landslide hazards?

The water increases the pore water pressure, effectively lifting the slide mass from stable ground beneath.

What is one reason that ash fall is such a significant hazard?

The weight of the ash can cause structural damage to buildings.

How do magnetic stripes on the ocean floor serve as evidence for seafloor spreading?

Their symmetry on either side of the mid-ocean ridge shows that new crust is created, then split.

The occurrence of the same fossils on different continents are attributed to what theory?

Theory of Plate Tectonics

Active Fault

There are a variety of definitions, but one is displacement along a fault in the past 10,000 years; another definition is multiple displacements in the past 35,000 years.

Magnitude-Frequency Concept

There is generally an inverse relationship between the magnitude of an event and its frequency. (example: the larger the flood, the less frequently it happens)

The drainage density on clay soils is likely to have what relationship to the drainage density on gravel soils?

There is greater density on clay soils than on gravel.

Major reasons cited by opponents to drilling in the Arctic National Wildlife Refuge (ANWR) include all of the following, except...

There is no oil beneath ANWR ********

You are a geologist working in the field, and you come across some layered sedimentary rocks. You notice that there is a layer of marine sediment that separates fluvial deposits above and below. You also notice that certain plant fossils present in the older fluvial deposit aren't present in the younger deposits above the marine layer. What do you conclude?

There was an extinction event at that location sometime around the deposition of the marine layer.

Why are point pollution sources less difficult to control than nonpoint sources?

They are discrete and often regulate.

Why are wetlands good places to treat or partially treat wastewater?

They can be man-made, they are inexpensive compared to traditional wastewater treatment plants, and wetland plants use nutrients considered to be pollutants.

What adverse effect do groins and jetties both have on coastal erosion?

They stop littoral transport of sand and starve downdrift areas of sand.

Why are ancient impact craters difficult to identify on Earth?

They typically have been eroded or filled with younger sediments.

Oceanic crust is ___________ and is composed of slightly __________ dense rocks than continental crust.

Thinner, More

Which is the correct series indicating what hazards, on average, has caused the most deaths per year in the United States (most to least)?

Tornado and Windstorm, Lightning, Flood, Hurricane

Convection

Transfer of heat involving movement of particles; for example, the boiling of water, in which hot water rises to the surface and displaces cooler water that moves toward the bottom.

Liquefaction

Transformation of water-saturated granular material from the solid state to a liquid state.

Greenhouse Effect

Trapping of heat in the atmosphere by water vapor, carbon dioxide, methane, and chlorofluorocarbons (CFCs).

S Wave (Shear Waves)

Travel only through solid materials. Cannot move through liquids. Speed through rocks, such as granite, is approximately one-half that of P waves. They produce up-and-down or side-to-side motion at right angles to the direction of wave propagation.

How can biotechnology be utilized to reduce environmental damage caused by mineral mining?

Treating acid mine drainage, bioassisted leaching, and treating contaminated water.

You are studying climate change in the American Southwest over the past 200 years. Which type of proxy measurement might you collect in this part of the world?

Tree ring data from ancient trees.

Hurricane

Tropical cyclone characterized by circulating winds of 100 km/hr or greater generated over an area of about 160 km in diameter; known as typhoons in the Pacific Ocean.

Which layer of the atmosphere is where the weather occurs?

Troposphere

How does urbanization affect flooding?

Urbanization causes increased runoff because less water infiltrates the ground.

Water Power

Use of flowing water such as in a reservoir to produce electrical power.

Point Source

Usually discrete and confined sources of air or water pollutants such as pipes that enter into a stream or river or stacks emitting waste from factories or other facilities into the atmosphere.

The thistle landslide, in the state of ________, was one of the most expensive landslides in US history.

Utah

All of the following statements about geothermal energy are true, except...

Utilization of geothermal energy causes minimal environmental impacts. *****

The _________ includes all Earth material above the water table.

Vadose Zone

The ________ disaster in Italy was caused by landslides entering the lake.

Vaiont Dam

Which is not a reason why vegetation is a significant factor in slope stability?

Vegetation improves soil nutrients which increases soil firmness.

Which of the following plays a role in the wastewater renovation and conservation cycle?

Vegetation, soil and rock type, topography, climate *******

Creep

Very slow downslope movement of rock or soil.

Ash Fall

Volcanic ash eruption that blows up into the atmosphere and then rains down on the landscape.

Why might global warming increase the magnitude and/or frequency of weather-related hazards?

Warmer ocean waters will channel more energy into the atmosphere. Many hazardous natural events are controlled in part by the amount of water in the system.

Why is karst topography a concern for groundwater quality?

Waste disposed in karst areas can easily infiltrate into the ground.

Runoff

Water moving over the surface of Earth as overland flow on slopes or stream flow; that part of the hydrologic cycle represented by precipitation or snowmelt that results in stream flow.

Off-stream Use

Water removed or diverted from its primary source for a particular use.

If water is so abundant on Earth, why are water resources such a high concern?

Water resources are not necessarily where humans need them and when they need them

____________ are areas that are either inundated by water or in which the ground is saturated to shallow depths at least a few days in most years.

Wetlands

Distant Tsunami

When an earthquake happens out in the ocean, the water above it rises into a dome and splits in two. This one goes out further to sea and can strike distant land if it makes it that far. Can travel thousands of kilometers.

Oil

When referring to energy resources, may also be known as petroleum or crude oil; a liquid hydrocarbon generally extracted from wells.

Local Tsunami

When the dome of water splits, the other wave goes towards the closes piece of land.

Regional Metamorphism

Wide-scale metamorphism of deeply buried rocks by regional stress accompanied by elevated temperatures and pressures.

The cyclic nature of plate tectonics, detailing the formation of an ocean basin and its eventual destruction, is called the _______.

Wilson Cycle

Which of the following does not influence wave size?

Wind Temperature

Storm Surge

Wind-driven oceanic waves, usually accompanying a hurricane.

All of the following are potential adverse effects associated with wind power, except...

Windmills produce radioactive waste that must be properly disposed of. ******

Longshore Sediment Transport

With respect to coastal processes, refers to the transport of sediment in the nearshore environment by wave activity.

Directivity

With respect to earthquake hazards, refers to the fact that during some moderate to large earthquakes the rupture of the fault is in a particular direction and the intensity of seismic shaking is greater in that direction.

Efficiency

With respect to energy resources, refers to designing and using equipment that yields more power from a given amount of energy, resulting in wasting less of the energy to the environment as heat.

Tidal Power

With respect to energy resources, refers to the useful conversion of tidal currents to produce electrical power.

Cogeneration

With respect to energy resources, the recycling of waste heat to increase the efficiency of a typical power plant or factory; may involve production of electricity as a byproduct from industrial processes.

Expansive Soil

With respect to engineering properties of soils, refers to soils that, upon wetting and drying, will alternately expand and contract, causing problems for foundations of buildings and other structures.

Continuity Equation

With respect to hydrology of rivers, refers to the equation that the discharge (Q) of flow is equal to the product of the cross-sectional area of flow times the velocity of the flow. Q = W x D x V W = width of low in meters D = depth of the flow inters V = mean velocity of flow in meters/second

Concentration Factor

With respect to mining of resources, the ratio of a metal's necessary concentration for profitable mining to its average concentration in Earth's crust.

Forecast

With respect to natural hazards, refers to an announcement that states that a particular event such as a flood is likely to occur at a particular time, often with some probability as to how likely the event is.

Precursor Events

With respect to natural hazards, refers to physical, chemical, or biological events that occur before an event such as a flood, earthquake, or volcanic eruption.

Disaster Preparedness

With respect to natural hazards, refers to the actions of individuals, families, cities, states, or entire nations taken before a hazardous event to plan for that event and to minimize losses.

Warning

With respect to natural hazards, the announcement of a possible disaster such as a large earthquake or flood that could occur in the near future.

Biotechnology

With respect to resource management, refers to use of organisms to assist in mining of ores or cleaning up of waste from mining activities.

In the example of the streams of Yellowstone National Park detailed in this chapter, which species is the keystone species?

Wolves

If we don't manage our water resources better we may be facing ___________.

a food shortage in the future

Which of the following is likely to encourage the formation of magma? a) A decrease in overlying pressure exerted on hot rock within the asthenosphere b) An increase in overlying pressure exerted on hot rock within the asthenosphere c) Removal of carbon dioxide from hot rock within the asthenosphere d) Removal of water from hot rock within the asthenosphere

a) A decrease in overlying pressure exerted on hot rock within the asthenosphere

Which of the following is the best explanation for how earthquakes cause tsunamis? a) Earthquakes shift the sea floor up or downward, causing water to be displaced. b) Earthquakes heat the ocean water, causing it to expand and move as a wave. c) Earthquakes shake the water, causing ripples to move outward in all directions. d) Earthquakes do NOT cause tsunamis. They are caused by material "splashing" into the water. e) Earthquakes cause a release of gas on the sea floor that causes water to be displaced.

a) Earthquakes shift the sea floor up or downward, causing water to be displaced.

Which of the following rock structures usually form when rocks are shortened by lateral compression? a) Folds b) Batholiths c) Inclusions d) Gneiss e) Unconformities

a) Folds

Which of the following is a type of instream water use? a) Hydroelectric power generation b) Thermoelectric power generation c) Consumptive use d) Irrigation

a) Hydroelectric power generation

Which of the following is a nonfoliated metamorphic rock? a) Marble b) Gneiss c) Slate d) Schist

a) Marble

Which volcano is characterized by highly viscous rhyolitic magma and has eruptions that are rare and extremely explosive? a) Mount Lassen, California b) Mauna Loa, Hawaii c) Mount Fuji, Japan d) Mount Saint Helens e) Springerville, Arizona

a) Mount Lassen, California

A flash flood does not consist of which of the following elements: a) Occurs in the lower parts of drainage basin b) Produced by intense rainfall of short duration c) Occurs in a relatively small area d) None of these

a) Occurs in the lower parts of drainage basin

Which type of lava flow is faster? a) Pahoehoe b) Aa c) Neither, they both move at the same speed.

a) Pahoehoe

Which of the following is a direct effect of a disaster? a) People killed or injured b) Taxes levied to finance recovery c) Emotional distress d) Donation of money or goods e) All of the above

a) People killed or injured

Which process is a more influential driving force of plate motion? a) Slab pull b) Ridge push

a) Slab pull

Which of the following would probably occur during the months after a catastrophic impact? a) Sunlight blocked and photosynthesis reduced or stopped b) Shock waves c) Fireball d) Huge mushroom cloud e) Forest fires

a) Sunlight blocked and photosynthesis reduced or stopped

Which of these is not one of the three rock laws? a) The Law of Intrusive Layers b) The Law of Crosscutting Relationships c) The Law of Original Horizontality d) The Law of Superposition

a) The Law of Intrusive Layers

Which of the following is a true statement about transform boundaries? a) The relative motion of plates at a transform boundary can cause earthquakes. b)The Himalayan Mountains were created by a transform boundary. c) Subduction zones occur at transform boundaries. d) Most transform boundaries occur within continents. e) All of the above are correct.

a) The relative motion of plates at a transform boundary can cause earthquakes.

Which of the following plate boundaries is characterized by two plates sliding past one another and neither creation, nor destruction, of the lithosphere? a) Transform plate boundary b) Divergent plate boundary c) Convergent plate boundary d) None of the above

a) Transform plate boundary

Near Earth objects (NEOs) are _______. a) asteroids or comets that either reside and orbit between Earth and the Sun or have orbits that intersect with Earth's orbit b) found in the Oort cloud c) found in the asteroid belt between Mars and Jupiter d) found in the Kuiper belt e) all of the above

a) asteroids or comets that either reside and orbit between Earth and the Sun or have orbits that intersect with Earth's orbit

All of the following are structural methods used to prevent floods, EXCEPT... a) channel restoration. b) flood walls. c) artificial levees. d) dams. e) channelization.

a) channel restoration.

All of the following statements are true, EXCEPT _______. a) meteorites are considered "dirty snowballs" because they have a rocky core surrounded by ice b) the light from a "shooting star" is produced by frictional heating of a meteor in the atmosphere c) comets contain water, and impacts with Earth through geologic time delivered the water that later formed our oceans by volcanic degassing d) as a comet is warmed by the Sun, ice vaporizes and dust and gas particles escape, producing the comet tail

a) meteorites are considered "dirty snowballs" because they have a rocky core surrounded by ice

All of the following statements about global water abundance are true, except _________. a) most of the Earth's water is suitable for human use b) most of Earth's water is found in the oceans c) water is plentiful on Earth d) water is often not available in the right place, at the right time, and in the right form

a) most of the Earth's water is suitable for human use

All of the following are types of artificial control used to control natural hazards, EXCEPT... a) prohibiting construction on a floodplain. b) construction of a retaining wall. c) construction of artificial levees. d) construction of a dam along a river. e) channelization of a river. f) a seawall constructed along the coast.

a) prohibiting construction on a floodplain.

A catastrophe is...

an event in which damages are such that recovery and/or rehabilitation is a long, involved process.

Vegetation influences landslides in part by...

anchoring the slope materials.

Which of the following scales can be used to compare earthquakes in terms of the severity of shaking and the resulting impact on people and society? a) Richter Magnitude Scale b) Modified Mercalli Scale c) Moment Magnitude Scale d) Ground Shake Scale

b) Modified Mercalli Scale

Which of the following are characteristics of a shield volcano? a) Magma with a high silica content, explosive b) Not explosive, built up from numerous lava flows c) Small volcano formed from tephra d) Cone-shaped, explosive

b) Not explosive, built up from numerous lava flows

Isostasy refers to which of the following? a) Earth's magnetic field has, at certain times, been reversed in the sense that its polarity was the opposite of what it is today. b) Thicker, more buoyant crust stands topographically higher than thinner, denser crust. c) The speed of seismic waves can vary, depending on what layer of Earth's interior they are passing through. d) None of the above

b) Thicker, more buoyant crust stands topographically higher than thinner, denser crust.

Which of the following is a type of climate proxy data? a) ancient bones b) tree rings c) historic legends d) soil composition

b) Tree Rings

Which of the following is an example of a natural service function of an ecosystem? a) The removal of vegetation due to wildfire causes soil erosion. b) Wetlands filter and clean polluted surface water. c) An earthquake exposes new rocks. d) All of the above are correct.

b) Wetlands filter and clean polluted surface water.

All of the following are uses for clays, EXCEPT... a) adobe. b) as a component in toothpaste. c) kitty litter. d) as a filler for paper. e) pottery.

b) as a component in toothpaste.

All of the following are precursor events that may help predict a landslide, earthquake, or volcano, EXCEPT... a) anomalous uplift. b) flooding. c) foreshocks. d) bulging or swelling of the ground surface. e) creep of the ground surface.

b) flooding.

All of the following are characteristics of a mineral, EXCEPT it... a) must have a characteristic chemical formula. b) must be hard enough to scratch glass. c) must normally be a solid. d) must be naturally formed. e) must have a characteristic crystalline structure in most cases. f) All of the above are characteristics of minerals.

b) must be hard enough to scratch glass.

All of the following statements are part of Luna Leopold's new water management philosophy, except ______. a) surface water should be used when it is available during wet years, and groundwater should be reserved for dry years b) we are much better prepared to handle water deficiencies than floods c) surface water and groundwater are both subject to natural flux d) excesses and deficiencies in water are natural and can be planned for e) groundwater can be pumped out at a rate exceeding the replenishment rate in dry years as long as we allow it to be replenished by natural discharge and artificial recharge during wet years

b) we are much better prepared to handle water deficiencies than floods

Which of the following minerals was created by weathering processes?

bauxite *****

To be considered a mineral, an element or chemical compound must __________.

be naturally formed

The balance of sediment input and output along a stretch of coastline is called the ______.

beach budget

______ is a "soft" solution to coastal erosion that tends to imitate natural processes.

beach nourishment

The dominant processes in an ecosystem are...

energy flows and chemical cycles.

Why does population growth result in more death and damage from natural hazards? a) Population increase puts more people at risk from a natural event. b) Population increase forces more people to settle in hazardous areas. c) Both of the above statements are true.

c) Both of the above statements are true.

Which of the following is the most violent type of volcanism? a) Volcanic dome eruption b) Shield volcano eruption c) Caldera-forming eruption d) Cinder cone eruption e) None of the above

c) Caldera-forming eruption

Which best describes Mount St. Helens and other Cascade volcanoes? a) Active shield volcanoes built by a continent-continent collision b) Shield volcanoes built as western North America moved over a hotspot deep in the mantle c) Composite volcanoes built on a continental margin above a sinking slab of oceanic lithosphere d) Composite volcanoes built by a mid-oceanic ridge

c) Composite volcanoes built on a continental margin above a sinking slab of oceanic lithosphere

Which of the following does not tend to increase biodiversity in an ecosystem? a) Evolution b) Diverse Habitats c) Extreme Disturbance d) Relatively constant environmental factors

c) Extreme Disturbance

Which of the following is a way to reduce the human footprint on the environment? a) Work to reduce species biodiversity on Earth so there is less completion for resources. b) Increase the human population so there are more people to care for the Earth. c) Force humans to use resources more efficiently. d) Generate more waste than other species. e) All of the above are correct.

c) Force humans to use resources more efficiently.

All of the following are potential effects of global warming, except... a) sea levels rising b) air pollution increasing c) growing seasons shortened d) extreme weather increasing e) none of the above

c) Growing Seasons Shortened

Which of the following statements is true? a) Rocks on continents are about the same age as the rocks on the ocean floor. b) Rocks on the ocean floor are often about twenty times as old as the oldest rocks on continents. c) Rocks on continents are in general older than rocks on the ocean floor. d) The oldest rocks on continents are about twice as old as the oldest rocks on the ocean floor. e) The oldest rocks on the ocean floor are about twice as old as the oldest rocks on continents.

c) Rocks on continents are in general older than rocks on the ocean floor.

Which of the following is the definition of a stream's gradient? a) The slope of the stream's banks b) The surface of the stream's cross-section c) The stream's vertical drop per unit of horizontal distance d) None of the above

c) The stream's vertical drop per unit of horizontal distance

All of the following are advantages or beneficial uses of wetlands, except _____. a) they provide a buffer for erosion b) they filter pollutants from water c) they almost always are areas of groundwater discharge and thus generally not susceptible to pollution d) they are often highly productive areas for wildlife

c) They almost always are areas of groundwater discharge and thus generally not susceptible to pollution

Which of the following is true of natural hazards? a) They could be prevented if we used proper engineering methods. b) They rarely cause great financial losses. c) They are natural processes that become hazards when people live or work in areas where they occur. d) All of the above are correct.

c) They are natural processes that become hazards when people live or work in areas where they occur.

Which of the following hazards has a high catastrophe potential? a) Expansive soils b) Frost and freezing weather c) Volcanic eruption d) Lightning e) Coastal erosion

c) Volcanic eruption

True/False: Water use in the US increased exponentially from 1960 to 1995.

false

Which of the following describes how wolves affect streamside vegetation in Yellowstone National Park? a) When wolves are present in the park, elk flock to the streams, denuding the vegetation. b) Wolves keep vegetation in check by grazing on it. c) When wolves are present in the park, elk avoid the challenging topography near streams, resulting in an increase in streamside vegetation. d) When there were no wolves in the park, streamside vegetation grew out of control.

c) When wolves are present in the park, elk avoid the challenging topography near streams, resulting in an increase in streamside vegetation.

All of the following are possible hypotheses used to explain the extinctions at the beginning of the Younger Dryas event, EXCEPT _______. a) overkill of megafauna by humans b) rapid climate cooling c) catastrophic caldera-eruption of volcano d) an extraterrestrial impact that occurred 12,900 years ago

c) catastrophic caldera-eruption of volcano

All of the following are hazards associated with ash fall, EXCEPT... a) destruction of vegetation, including crops and trees. b) health hazards, such as irritation of the respiratory system and eyes, caused by contact with the ash. c) massive asphyxiation of people and animals in the area. d) "flame out" of engines of jetliners ""as melted silica-rich ash forms a thin coating of volcanic glass in the engines. e) structural damage to buildings, caused by the increased load on roofs. f) contamination of surface water by sediment.

c) massive asphyxiation of people and animals in the area.

All of the following statements about the K-T boundary are true, EXCEPT _______. a) the K-T boundary coincides with an extinction event that brought an end to the dinosaurs b) the K-T boundary indicates that there was an extraterrestrial impact 65 million years ago c) the K-T boundary coincides with the extinction of the Clovis culture of Paleo Americans d) the K-T boundary contains an iridium anomaly

c) the K-T boundary coincides with the extinction of the Clovis culture of Paleo Americans

Snow avalanches tend to move down tracks called ________.

chutes

The largest particle a river can transport is called its _______ and the total load of sediment that a river carries in a given time is called its _______.

competency, capacity

Pumping of groundwater from a well creates a(n) ______ in the aquifer.

cone of depression

A(n) ______ aquifer is largely isolated from the Earth's surface above it.

confined

The Tunguska event was determined to be a giant airburst because there was no _______.

crater

Intrusive igneous rocks differ from extrusive igneous rocks primarily in their ____________.

crystal sizes

Which of the following geologic factors is likely to increase biodiversity? a) The presence of harsh environments, such as hot springs or nutrient-poor rocks and soil b) Moderate amounts of disturbance, such as wildfires, violent storms, or volcanic activity c) Relatively constant environmental factors such as temperature, precipitation, and elevation d) All of the above

d) All of the above

Which of the following is a natural service function of ecosystems? a) Protection from flooding b) Protection from landslides c) Protection from erosion d) All of the above

d) All of the above

Which of the following provides evidence for the theory that today's continents were all once part of a large supercontinent? a) The distribution of animal fossils on various continents b) The existence and direction of ice flow of glaciers on various continents c) The distribution of plant fossils on various continents d) All of the above

d) All of the above

Which of the following is a reason why environmental problems may be difficult to solve? a) Environmental problems involve the possibility of irreversible change. b) Expediential growth of the problem is often encountered. c) There is often a lag time between when a change occurs and when it is recognized as a problem. d) All of the above are correct.

d) All of the above are correct.

Which of the following is an example of how human activities are likely to increase the impact of natural disasters? a) Removal of coastal wetlands and conversion of land to urban areas reduces natural protection from storm waves. b) Deforestation and conversion of land to agriculture yield more frequent flooding events. c) Developing an agricultural center on the fertile flank of an active volcano places a greater number of people at risk. d) All of the above are correct.

d) All of the above are correct.

Which of the following is true about human domination of ecosystems? a) Human-driven processes are resulting in the reduction of biodiversity. b) The most significant factor that has led to human domination of Earth's ecosystems is land transformations. c) It has resulted in disastrous conditions in some areas. d) All of the above are true e) None of the above is true.

d) All of the above are true

The global water cycle involves which of the following processes? a) Water evaporates from the ocean into the atmosphere. b) Water flows into the oceans. c) Water precipitates from the atmosphere onto land. d) All of the above are true.

d) All of the above are true.

What is being done to minimize the risk posed by asteroids and comets? a) We are developing the technology to nudge an asteroid out of our path. b) We are identifying and tracking objects that cross Earth's orbit. c) We are considering evacuation for small events. d) All of the above are true.

d) All of the above are true.

Which of the following is true of karst topography? a) Karst topography can result in diversion of surface water to subterranean routes. b) Sinkholes often form in karst regions. c) Karst topography occurs in areas underlain by limestone. d) All of the above are true.

d) All of the above are true.

Paleomagnetism can help scientists determine... a) The age of oceanic crust. b) Earth's tectonic history. c) The rate of seafloor spreading. d) All of the above.

d) All of the above.

Which of the following regions has the least risk from tsunamis? a) Hawaii b) Japan c) Northeastern coast of the Indian Ocean d) Atlantic coast of the United States e) Pacific Northwest coast of the United States

d) Atlantic coast of the United States

True/False: Wetlands contribute to poor water quality and harm flood control.

false

Which earthquake would cause the most damage, given that all other factors are the same? a) Magnitude 7.5, focus depth 15 km b) Magnitude 4.5, focus depth 20 km c) Magnitude 7.5, focus depth 20 km d) Magnitude 7.5, focus depth 10 km e) Magnitude 4.5, focus depth 10 km

d) Magnitude 7.5, focus depth 10 km

Based on your knowledge of the magnitude-frequency concept, which would you expect to do the most work altering the landscape in a given area? a) Large hurricanes b) Tornadoes c) Light spring rains d) Moderate floods of an area river

d) Moderate floods of an area river

Which of the following statements is true about extrusive igneous rocks? a) They have cooled and crystallized slowly at depth. b) They are usually coarse-grained. c) inclusions within sedimentary rocks. d) They are usually fine-grained.

d) They are usually fine-grained.

All of the following are factors that affect the damage caused by floods, EXCEPT... a) land use on the floodplain. b) the depth and velocity of the water and frequency of flooding. c) rate of rise and duration of flooding. d) abundance of fish in the water. e) sediment load deposited. f) effectiveness of forecasting, warning, and emergency systems

d) abundance of fish in the water.

All of the following are examples of the environmental degradation experienced by the Kissimmee River prior to restoration, EXCEPT... a) adjacent wetlands with populations of birds and fish were drastically reduced. b) channelization of the river. c) meanders removed and river straightened. d) acidification of the river. e) draining of the floodplain.

d) acidification of the river.

What are factors contributing to the decline in coral reefs around the globe? a) the warming of the oceans b) overfishing c) pollution d) all of the above

d) all of the above

All of the following hazards are linked to hurricanes, EXCEPT... a) landslides. b) lightning. c) coastal erosion. d) earthquakes. e) flooding.

d) earthquakes.

All of the following are common rock-forming minerals, EXCEPT... a) quartz. b) mica. c) feldspar. d) granite. e) calcite.

d) granite.

All of the following are ways to minimize the hazard from tsunamis, EXCEPT... a) stricter building codes for structures on vulnerable coastlines. b) greater emphasis on detection and warning systems. c) increased planting of native vegetation and preservation of coastal wetlands to serve as a buffer for breaking waves and tsunamis. d) making sure all people can either run or swim. e) educating people about tsunami indicators and what to do during a tsunami.

d) making sure all people can either run or swim.

All of the following are used to forecast volcanic eruptions, EXCEPT... a) monitoring hydrologic and thermal changes. b) topographic (slope) monitoring. c) monitoring seismic activity. d) monitoring local weather changes. e) monitoring volcanic gas emissions.

d) monitoring local weather changes.

All of the following responses to a hazard would be considered anticipatory, EXCEPT... a) evacuation. b) disaster preparedness. c) insurance. d) search and rescue. e) land-use planning.

d) search and rescue.

All of the following are signs that an earthquake might occur soon in a given area, EXCEPT... a) the water level is visibly lower than usual in a harbor. b) an increase in radon gas is detected. c) small earthquakes have been felt for several days. d) there is a landslide.

d) there is a landslide.

All of the following are methods for reducing water consumption, except _______. a) water conservation through the use of more efficient appliances b) more efficient agricultural irrigation methods c) water recycling by industrial companies d) use of alternative water supplies e) none of the above

d) use of alternative water supplies

Which of the following does not influence wave size? a) wind velocity b) wind duration c) wind fetch d) wind temperature

d) wind temperature

As a tsunami nears land, the water depth decreases, causing the tsunami to slow down. This decrease in velocity causes a(n) _______ in the spacing between the wave crests, also known as the wavelength.

decrease

Hurricanes develop from areas of low atmospheric pressure called tropical ______.

depressions

The tsunami warning system in the region of the 2004 tsunami __________.

did not exist

Which of the following is a method of flood-proofing? a) Constructing flood walls or earth berms around buildings to seal them from floodwaters b) Installing improved drains with pumps to keep flood waters out c) Raising the foundation of a building above the flood hazard level d) Using waterproofing construction such as waterproofed doors, waterproofed basement walls and windows e) All of the above

e) All of the above

Which of the following is a suggested hypothesis for mass extinction events? a) Extremely large volcanic events b) Rapid climate change c) Plate tectonics d) Impact of a large extraterrestrial object e) All of the above

e) All of the above

he 1980 eruption of Mt. St. Helens included which of the following? a) A lateral blast b) Mudflow c) Pyroclastic flow d) Debris avalanche e) All of the above

e) All of the above

How might climate change affect natural hazards? a) Global warming will likely increase the frequency and severity of hazardous weather. b) Global warming will increase the incidence of precipitation in certain regions and thus subsequent flooding. c) Rising sea level due to climate change will lead to an increase in coastal erosion. d) Global warming will increase the occurrence of droughts in certain regions, and deserts are likely to expand. e) All of the above are correct.

e) All of the above are correct.

Which of the following is a possible response to earthquake hazards? a) Increase insurance and relief measures. b) Design buildings to withstand at least moderate shaking. c) Site critical facilities such as schools, hospitals, and police stations away from active faults or sensitive earth materials. d) Develop earthquake warning systems. e) All of the above are correct.

e) All of the above are correct.

Which of the following is an environmental effect of dams? a) Downstream of the reservoir, rivers and coasts are deprived of sediment, encouraging river erosion and coastal erosion on beaches. b) Land, with all its resources, is lost in the area flooded by the reservoir. c) Ecosystems are fragmented, as migrating fish are blocked from moving upstream of the dam. d) The reservoir traps sediment that is transported from upstream by streams that enter the reservoir. e) All of the above are true.

e) All of the above are true.

Which of the following signs should cause a person to leave a beach or low-lying area? a) The ocean begins to recede, showing the ocean floor. b) A giant wall of water is seen moving toward the beach. c) A tsunami warning siren is heard. d) An earthquake is felt near a susceptible coastline area. e) All of the above should cause a person to move to higher ground.

e) All of the above should cause a person to move to higher ground.

All of the following are detrital (clastic) sedimentary rocks, EXCEPT... a) sandstone. b) siltstone. c) schist. d) conglomerate. e) gypsum.

e) gypsum.

All of the following are true about the 2004 Indonesian Tsunami, EXCEPT... a) the total length of the rupture zone was about the length of the state of California. b) over three-quarters of the deaths that occurred were in Indonesia. c) in Thailand, elephants started trumpeting about the time of the earthquake and became agitated prior to the tsunami's landfall. d) the earthquake that caused the 2004 Indonesian Tsunami occurred along a subduction zone. e) had the Indian Ocean Tsunami Warning System not worked, the death toll would have been higher.

e) had the Indian Ocean Tsunami Warning System not worked, the death toll would have been higher.

All of the following are effects of tsunamis, EXCEPT... a) contaminated water supplies. b) coastline erosion. c) disease. d) fires from ruptured gas lines. e) increased volcanic activity.

e) increased volcanic activity.

All of the following are considered wetlands, except _______. a) swamp b) marsh c) vernal pool d) bog e) permafrost

e) permafrost

All of the following are stages of the earthquake cycle, EXCEPT... a) seismic inactivity. b) increased seismicity, following a period of inactivity. c) the major earthquake. d) foreshocks. e) seismic readjustment after rupture.

e) seismic readjustment after rupture.

All of the following factors affect the severity of the shaking felt during an earthquake, EXCEPT... a) the direction of the fault rupture. b) the magnitude of the earthquake. c) local soil and rock conditions. d) the frequency of the seismic waves. e) the number of foreshocks.

e) the number of foreshocks.

The noncohesive downslope movement of saturated Earth materials is called a(n) ________.

earthflow

The process of altering a site or area with the objective of reestablishing indigenous, historical ecosystems is known as...

ecological restoration.

A(n) ______ stream gains water from an aquifer.

effluent

Reversals in Earth's magnetic field are apparently random, but occur on average...

every few hundred thousand years.

How might the 1983 Thistle landslide have been prevented?

examination of landslide history and installation of drains

In an area of crustal _______ you can expect to find _______ faults.

extension, normal

What regions of the world are at risk for volcanic eruptions? a) Italy b) Mexico c) Alaska d) Japan e) Pacific Northwest's Cascade Range f) All of the above

f) All of the above

Which of the following gases is/are emitted during volcanic activity? a) Sulfur dioxide b) Carbon monoxide c) Water vapor d) Carbon dioxide e) Hydrogen sulfide f) All of the above

f) All of the above

Which of the following may be used to predict earthquakes? a) Anomalous animal behavior b) Emission of radon gas c) Seismic gaps along faults d) Pre-seismic deformation of the ground surface e) Patterns and frequency of earthquakes, or foreshocks f) All of the above

f) All of the above

In order for a community to be tsunami ready, it must... a) have ways to alert the public. b) promote a community awareness program to educate people concerning a tsunami hazard. c) have ways to receive tsunami warnings from the National Weather Service or other agencies. d) establish an emergency operation center with 24-hour capability. e) develop a tsunami preparedness plan with emergency drills. f) do all of the above.

f) do all of the above.

All of the following processes are part of the rock cycle, EXCEPT... a) melting. b) weathering. c) deposition. d) metamorphism. e) lithification. f) radiation.

f) radiation.

All of the following are earthquake effects, EXCEPT... a) landslides. b) fires. c) shaking of the ground. d) disease. e) liquefaction. f) thunderstorms.

f) thunderstorms.

True/False: A cone of depression forms when water is pumped from an unconfined aquifer, but not when the water is pumped from a confined aquifer.

false

True/False: A safety factor of 1 implies that driving forces are 10 times greater than resisting forces.

false

True/False: Assateague Island, Maryland, has avoided coastal erosion because it is largely undeveloped.

false

True/False: Concerns over water resources focus on the lack of abundance of water on Earth.

false

True/False: Groundwater in karst systems is very well protected from pollution.

false

True/False: Groundwater mining in the oglala aquifer is a function of the very small amount of water contained in the aquifer.

false

True/False: Groundwater movement is proportional to porosity.

false

True/False: Hillslopes typically are static systems.

false

True/False: Hurricane waves at sea are the greatest hazard presented by hurricanes.

false

True/False: Land subsidence in te Central Valley, California, results mostly from oil extraction.

false

True/False: Logging operations always increase landslide hazards.

false

True/False: Ocean swell consists of a chaotic distribution of wave heights and lengths.

false

True/False: Refraction occurs because waves speed up as they reach the shallower water near the beach.

false

True/False: Runoff in areas of shale bedrock tends to be less that in areas of gravel because of the high porosity of shale.

false

True/False: Sea-cliff erosion tends to occur at a greater rate during the summer.

false

True/False: Slumps and debris flows tend to occur separately in time and space.

false

True/False: Soft stabilization of coastal erosion utilizes seawalls constructed of soft, cushions materials versus hard materials such as steel.

false

True/False: The Cape Hatteras lighthouse was moved inland because a major storm damaged it in 1994.

false

True/False: The Vaiont Dam disaster occurred when landslides caused the dam to fail.

false

True/False: The cost of completely stabilizing the La Conchita, California, landslide is far less than the cost of relocating residents to more stable ground.

false

True/False: The expense of preventing landslides is usually too great for the prevention strategies to be feasible.

false

True/False: The water table separates confined and unconfined aquifers.

false

Some metamorphic rocks are potential environmental hazards because...

foliation can represent a plane of weakness in the rock.

In Yosemite National Park, strong bedrock influences the formation of what types of hillslopes?

free face and talus

On what information is a landslide hazard map based?

geology and inventory of past landslides

An active fault is defined as a fault that...

has experienced movement during Holocene time.

When an impact crater is formed, the shock can metamorphose rocks in the impact area. Most of the metamorphism is the result of _______.

high pressure

Acceptable risk is the risk we are willing to endure in a given situation. Most of us agree that the acceptable risk of driving in a car is _______ whereas the acceptable risk from a nuclear power plant is _______.

high, low

A meteorite impact crater can be distinguished from a volcanic crater through the presence of...

high-pressure metamorphic minerals.

The ability of an Earth material to transmit water is called its ______.

hydraulic conductivity

The global ______ involves the transfer of water between Earth's water reservoirs.

hydrological cycle

The majority of the Moon's craters were formed by _______.

impact

Saltwater intrusion occurs ___________.

in coastal areas when salty groundwater forms a cone of ascension in response to a loss of overlying freshwater

Where do steelhead trout of southern California spawn?

in mountain streams

A(n) ______ stream typically lies above the water table.

influent

A pair of ______ typically protects the channel of a river entering the sea.

jetties

Species with disproportionate influences on their ecosystems are called

keystone species.

Most of the energy of impact is in the form of _______, or energy of movement. This energy is transferred to Earth's surface through a _______ that propagates into Earth.

kinetic energy, shock wave

A volcanic mudflow that occurs when ash and other pyroclastic materials become saturated with water is called a(n)...

lahar.

The _______ a flood is, the _______ frequently it is likely to occur.

larger, less

In 1973, people of the town of Vestmannaeyjar, on the Icelandic island of Heimaey, attempted to control _______ from a nearby erupting volcano using _______.

lava flows, hydraulic chilling

______ is the combination of beach drift and longshore drift.

littoral transport

On July 12, 1993, a M 7.8 earthquake in the Sea of Japan produced a tsunami that extensively damaged Anoae, a small town on the southern tip of the island. There was virtually no warning because the huge waves arrived only 2 to 5 minutes after the earthquake, killing 120 people. This tsunami is best classified as a...

local tsunami.

________ and ________ can severely increase landslide hazards in mountainous terrains.

logging, road construction

The current flowing parallel to and just offshore of a beach is called...

longshore current

Molten rock, before it is erupted onto Earth's surface, is called _______.

magma

Hurricanes in the tropical Atlantic Ocean tend to originate as...

major pressure lows over the equatorial ocean.

All slope movements fall under the term ____________.

mass wasting

When an earthquake hits an area constructed on silt and clay, ground shaking is greatly increased. This phenomenon is called _______.

material amplification

Tsunamis can be detected on the open ocean through __________.

measurement of changes in pressure exerted on the sea bottom by the water column

A meteoroid that enters Earth's atmosphere is called a __________.

meteor

When a celestial object actually strikes the surface of Earth, the object is called a(n) _______.

meteorite

Volcanism occurring at _______ produces _______.

mid-ocean ridges, basaltic rocks

Tsunamis travel...

more slowly in shallow water than in deep water, causing wave crests to rise.

Seawalls result in _______ beaches and a(n) _______ in biodiversity.

narrower, decrease

Coastal marshes reduce flooding and coastal erosion. This impact is an example of...

natural service functions of ecosystems.

It is believed that the asteroid that caused the K-T extinction struck Earth _______.

on the Yucatan Peninsula

What factor has contributed to the drying up of streams and the intrusion of ally groundwater in the Long Island area?

overpumping of groundwater

Beach erosion on Great Lakes shorelines is exacerbated by...

periodic high lake levels

Plastic deformation results in...

permanent change in the shape of the rocks.

A submarine trench is always associated with...

subduction.

As an ecosystem evolves following disturbance, it follows the path of _________.

succession.

As waves are sorted as distance increases from the source area, they develop a regular frequency and become ______.

swell

The E-10 zone in the coastal erosion hazard mapping scheme is the zone in which erosion is predicted to happen over the next ______ time period.

ten year

Why has water quality degraded in the Edwards aquifer?

the aquifer is recharged by rivers flowing into karst limestones

How did geology influence the Vaiont Dam disaster?

the geology in the adjacent hillslopes was dominated by weak rocks

The Mohorovicic discontinuity represents...

the boundary between mantle and crust.

A mega-tsunami is a wave that is about 100 times higher than the largest tsunami produced by an earthquake. Mega-tsunamis are very rare and are produced by..

the impact of large asteroids.

The base level of a stream is...

the lowest level to which a river may erode.

Tsunamis can take several hours to cross the ocean because __________.

the ocean is too deep to transmit the waves

The degree of urbanization consists of...

the percentage of impervious cover and the percentage of area served by storm sewers.

At convergent boundaries...

the plate of lower density subducts into the mantle.

True/False: A rise in sea level is likely to cause greater coastal erosion.

true

True/False: As waves enter the coastal zone, wavelengths shorten and wave heights increase.

true

True/False: Climate influences landslides by determining the vegetation present on a slope.

true

True/False: Coastal landscapes are capable of rapid change because continental and oceanic processes converge there.

true

True/False: Debris avalanches tend to occur on steep slopes.

true

True/False: Desalination is used only where alternative water sources are unavailable.

true

True/False: Erosion of the base of a slope may increase landslide hazards.

true

True/False: Grading of slopes can either increase or decrease landslide hazards.

true

True/False: Hillslopes in weaker rocks tend to be more gently sloped.

true

True/False: Hurricanes tend to strengthen over warn ocean water.

true

True/False: Hydroelectric water use is considered non-consumptive.

true

True/False: Littoral transport is driven by the oblique approach of waves to the shoreline.

true

True/False: Sinkholes often result from water table fluctuations.

true

True/False: The goal of groin installation is to trap sand.

true

True/False: The velocity and depth of surface water flow influence the amount of soil or sediment the water can erode.

true

True/False: Water management in the Colorado River basin has implications both for human use of water and for river ecology in the Grand Canyon.

true

True/False: Water pressure is generally greater in a confined aquifer than in an unconfined aquifer.

true

True/False: Lake level variations are the strongest control on erosion along the Great Lakes shorelines.

true`

A _______ is a sensor on the bottom of the ocean that detects small changes in the pressure exerted by the increased volume of water as a tsunami passes overhead.

tsunameter

Transform plate boundaries are marked by...

two plates sliding relative to one another.

Tropical cyclones are called ______ in the western Pacific Ocean.

typhoons

Tsunamis are generated by __________.

vertical displacement of ocean water

______ is related to the amount of water necessary to produce a product.

virtual water

One of the most effective methods of stopping an active slide is ________.

water drainage

______ are areas that are either inundated by water or in which the ground is saturated to shallow depths at least a few days in most years.

wetlands

True/False: A closed system is not amenable to scientific scrutiny.

False

True/False: Because human population is becoming quite large, the doubling time for population has decreased.

False

True/False: Ducktown, Tennessee, suffered extreme environmental modification due to a large earthquake in 1811.

False

True/False: Earth systems science depends on the validity of the Gaia hypothesis.

False

True/False: The approach of Earth systems science maintains that application of technological systems is the best means to address environmental degradation.

False

True/False: The health of the Aral Sea has become compromised by extensive aquatic pollution.

False

True/False: The scientific method begins with the development of a theory, which is then tested.

False

The law of ________ states that rocks with similar fossils are most likely of a similar geologic age.

Faunal Succession

Seafloor Spreading

The plate tectonics concept that new crust is continuously added to the edges of lithospheric plates at divergent plate boundaries as a result of upwelling of magma along mid-oceanic ridges.

Isostasy

The principle stating that thicker, more buoyant crust is topographically higher than crust that is thinner and denser. Also, with respect to mountains, the weight of rocks of the upper crust is compensated by buoyancy of the mass of deeper crystal rocks; that is, mountains have "roots" of lighter crustal rocks extending down into the denser mantle rocks, like icebergs in the ocean.

Geology

The science of Earth, including its structure, composition, and history.

Asthenosphere

The upper zone of Earth's mantle, located directly below the lithosphere; a hot, slowly flowing layer of relatively weak rock that allows for the movement of the tectonic plates.

A(n) ________ is a hypothesis that has withstood testing through a sufficient number of experiments.

Theory

________ growth implies that a constant percentage of humans are added each year.

Exponential

The ________ suggests that scientific certainty is not necessary in order to address a serious environmental problem.

Precautionary Principle

What are the 5 fundamental concepts that establish a philosophical framework for our investigation of environmental geology?

1. The increasing world population is the number-one environmental problem. 2. Sustainability is the preferred solutions to many environmental problems. 3. Having an understanding of the Earth system and rates of change in systems is critical to solving environmental problems. 4. Earth processes that are hazardous to people have always existed. These natural hazards must be recognized and avoided when possible, and their thereat to human life and property minimized. 5. Results of scientific inquiry to solve a particular environmental problem often result in a series of potential solutions consistent with the scientific findings. Which solution we choose reflects our value system.

Sustainability

A difficult term to define but generally refers to development or use of resources in such a way that future generations will have a fair share of Earth's resources and inherit a quality environment. In other words, sustainability refers to types of development that are economically viable, do not damage the environment, and are socially just.

Plate Tectonics

A model of global tectonics that suggests that the outer layer of Earth, known as the lithosphere, is composed of several large plates that move relative to one another; continents and ocean basins are passive riders on these plates.

Environmental Unity

A principle of environmental studies that states that everything is connected to everything else.

Growth Rate

A rate usually measured as a percentage by which something is changing; for example, if you earn 5 percent _interest in a bank account per year, then the growth rate is 5 percent per year.

Gaia Hypothesis

A series of hypotheses that explain how Earth as a system may operate with respect to life. Metaphorically, Earth is viewed as a giant organism consisting of various interactive systems with distinct feedback and thresholds that result in producing an environment beneficial to the many life forms on Earth. Furthermore, life is an important ingredient in producing that environment.

What is the Gaia hypothesis?

A set of hypotheses that likens Earth to a superorganism with interrelated, mutually adjusting systems.

Theory

A strong scientific statement. A hypothesis may become a theory after it has been tested many times and has not been rejected.

Environmental geology is particularly concerned with ________ geologic time.

Recent

System

Any part of the universe that is isolated in thought or in fact for the purpose of studying or observing changes that occur under various imposed conditions.

Environmental Geology

Application of geologic information to environmental problems.

What geological era are we living in now?

Cenozoic

Input-Output Analysis

Important method for analyzing change in open systems.

Why is human population growth often considered the foremost environmental problem?

Increasing population strains resources and creates additional wastes.

________ analysis evaluates change in open systems by examining the relative rates at which materials enter and leave a system.

Input-Output

Magnetic Reversal

Involves the change of Earth's magnetic field between normal polarity and reverse polarity; also sometimes known as geomagnetic reversal.

Why is the Gaia hypothesis not a theory?

It has not withstood a sufficient number of experiments to test it.

Continental Drift

Movement of continents in response to seafloor spreading; the most recent episode of continental drift supposedly began about 200 million years ago with the breakup of the supercontinent Pangaea.

Humans evolved during the ________ Epoch of geologic time.

Pleistocene

________ is helping natural hazards to become catastrophes.

Population Increase

How are the impacts of natural hazards linked to population growth?

Population growth concentrates people and resources, such that the impacts of an individual natural hazard can be greater.

"Absolute scientific proof of an environmental problem is not possible, so we should take action anyway. It is better to be safe than sorry." This quote is an example of...

The Precautionary Principle

Average Residence Time

The amount of time it takes for the total stock or supply of material in a system to be cycled through the system.

What is average residence time?

The average time required for the total stock of a material to be cycled through a system.

What is the average residence time?

The average time required for the total stock of a material to be cycled through a system.

Moho

The boundary between the crust and mantle, also known as the Mohorovicic discontinuity; distinguished by compositional differences between the rocks of the crust and the mantle.

Scientific Method

The method by which scientists work, starting with the asking of a question concerning a particular problem, followed by development and testing of hypotheses.

Crust

The outermost layer of the solid Earth, embedded in the top of the lithosphere, that varies in thickness from 6 to 7 km below the oceans to as much as 70 km beneath continental mountain ranges.

Geological Time

Time extending from the beginning of Earth to the present; determined in part from Earth's history as recorded in the rocks and sediments that have been deposited and formed at various times; the geologic time scale is the chronological arrangement of rocks of various ages, generally from the oldest event to the youngest.

True/False: The concept of a land ethic maintains that we are responsible to the entire environment, not just to humans.

True

True/False: The concept of environmental unity states that one action causes subsequent actions in linked systems.

True

________, a principle proposed by James Hutton in 1785, is often summarized in the phrase, "the present is the key to the past."

Uniformitarianism

What has caused the Aral Sea to shrink dramatically over the past 40 years?

Water diversions in streams feeding the sea.

Core

With respect to the interior of Earth, the central part of Earth below the mantle, divided into a solid inner core with a radius of approximately 1300 km and a molten outer core with a thickness of about 2000 km; the core is thought to be metallic and composed mostly of iron.

A landfill system is best described by which of the following? a) The input is greater than the output. b) The input equals the output. c) The input is less than the output.

a) The input is greater than the output.

Which of the following statements is an example of the principle of environmental unity? a) If an earthquake caused certain changes a million years ago, a similar earthquake would have caused similar changes 200 million years ago. b) Recycling and reusing are sustainable behaviors that help our environment. c) The diversion of a river for agriculture may cause a lake to dry up, which may cause the death of many organisms. d) If two rocks contain similar fossils, then the rocks are close to the same geologic age.

c) The diversion of a river for agriculture may cause a lake to dry up, which may cause the death of many organisms.

Which of the following is an example of uniformitarianism? a) A chemical fertilizer used by a farmer kills fish in a lake 200 miles away. b) Life helps regulate the planetary environment. c) We can assume that the processes of wind and water erosion observed today worked similarly two hundred million years ago. d) If two rocks contain similar fossils, then the rocks are close to the same geologic age.

c) We can assume that the processes of wind and water erosion observed today worked similarly two hundred million years ago.


Conjuntos de estudio relacionados

Psychology 1000 Final Exam Practice

View Set

Ch. 5 Investigating God's World A Beka Book 5th Grade

View Set

Life Insurance Policies and Annuity Contracts

View Set

Chapter 1 Lesson 3 Measure- A Common Language Review

View Set

STUDY GUIDE # 4 VALUES, ETHICS and ADVOCACY

View Set

Deferred Compensation Plan & Executive Bonus Plan

View Set

History 2 Exam 2 Essay Questions

View Set